EXAM MASTER MSK
A 17-year-old football player (punter) was tackled and forcefully slammed onto his right dominant shoulder 2 hours ago during a game. He had immediate pain, but it has subsided; he now experiences the same level of pain when he carries a heavy object. He wants to continue punting. He has full active and passive range of motion, but some pain (4/10) with abduction. There is no obvious deformity, and the skin over the shoulder is intact and not tented. There is no crepitus on palpation while moving the arm. Most of his tenderness seems to be on the superolateral right shoulder. What recommendation should be made to this patient? 1 "You can continue playing, as this is a minor injury." 2 "You need to see an orthopedic surgeon immediately." 3 "You can continue to play if you are pain-free." 4 "You will need a splint and therefore cannot play." 5 "You can continue to play after a cortisone injection."
Correct Answer: "You can continue to play if you are pain-free." Patients with a Type 1 acromioclavicular separation may resume playing as pain allows, but typically that takes 4 weeks. The patient will run a risk of further injury if he continues to play through the pain and convert a Type 1 into a Type 2 or worse.
A 53-year-old woman presents for an annual examination. She has a history of asthma for which she takes inhaled steroids and ß-agonists. She has no history of bone fractures and no family history of osteoporosis. She exercises regularly. Her menses used to be regular, but have just started to show some irregularity. She believes she might be entering menopause and asks for advice to prevent osteoporosis. Question Highlights What treatment will be recommended? 1 Hormone replacement therapy 2 1200 mg of calcium and 800 IU vitamin D daily 3 800 mg of calcium daily 4 A 75 mg tablet of risedronate weekly 5 200 units of nasal salmon calcitonin daily
Correct Answer: 1200 mg of calcium and 800 IU vitamin D daily The patient has no risk factors for osteoporosis. Even though she has noticed some irregularity in her menses, she does not have any other symptoms characteristic of entering menopause. She takes inhaled steroids, which are not recognized as a cause of increased bone loss in adults (as is the case for oral steroids). 1200 mg of calcium and 800 IU vitamin D daily are the recommended doses for adults, and regular exercise is recommended. 800 mg of calcium is incorrect.
A 33-year-old Caucasian man with a history of severe type 1 diabetes presents with a red and swollen pinky toe on his left foot. Further details from the patient include that he was running to answer his phone at home 2 days ago and stepped on an exposed carpet tack. He washed it out and covered it as soon as he could, but he has noted a significant increase in redness and severe restriction of movement of this toe, worsening significantly over the last 12 hours. He admits to "feeling feverish" for the past day and experiencing significant night sweats the previous night. He denies formally measuring his temperature. He has tried to take acetaminophen 500 mg 2 tablets every 8 hours without any noticeable relief. Pertinent physical examination findings include an oral temperature of 102.8°F, significant erythema and edema extending the entire fifth toe of the left foot, and restricted range of motion due to tenderness and swelling. There is a small amount of purulent drainage at the site of the wound. Question Highlights Based on the most likely diagnosis, how long will this patient need to be on antibiotics? 1 5 days 2 1 week 3 10 days 4 2 weeks 5 6 weeks
Correct Answer: 6 weeks
A 17-year-old high school football player was tackled and forcefully slammed onto his right dominant shoulder 2 hours ago during a game. He had immediate pain but was able to continue playing. He has full active and passive range of motion but some pain (4/10) with abduction. There is no obvious deformity, and the skin over the shoulder is intact and not tented. There is no crepitus on palpation while moving the arm. Most tenderness seems to be on the superolateral right shoulder. Question Highlights What is the most likely diagnosis? 1 Clavicle fracture 2 Sternoclavicular dislocation 3 Adhesive capsulitis 4 Glenohumeral dislocation 5 Acromioclavicular separation
Correct Answer: Acromioclavicular separation This patient is likely presenting with a grade I acromioclavicular (AC) separation. A fall directly onto the shoulder can result in either an AC separation or a clavicle fracture. Since this patient had no deformity or crepitus and was able to continue playing, a clavicle fracture is very unlikely. Movement or activity (continuing to play) would cause considerable pain.
An 18-year-old woman presents to the emergency department with her mother due to pain in her right leg after a car accident. She points to her lower leg and describes the pain as severe. Upon physical exam, there is extreme pain with passive movement of right leg with diminished sensation. When asking the patient to stand on the leg, she reports weakness and extreme tenderness. Upon use of the Stryker IC pressure monitor system, the patient's pressure was 35 mm Hg. Question What is the most likely diagnosis? 1 Acute compartment syndrome 2 Carpal tunnel syndrome 3 Exertional compartment syndrome 4 Rhabdomyolysis 5 Stress fracture
Correct Answer: Acute compartment syndrome The clinical picture is suggestive of acute compartment syndrome because the patient complained of pain and pressure. Additionally, the patient has a pressure of 35 mm Hg on the Stryker IC pressure monitor, which helps confirm compartment syndrome and indicates a need for intervention.
A 25-year-old man presents with back pain and stiffness. He states that he has had longstanding issues with back pain, although he denies any trauma to his back. He has noticed an increasing presence of stiffness and general fatigue along with the pain. He feels that these issues have gradually worsened over the last several months, and they are more persistent recently. The pain is much worse first thing in the morning, and he rates it a 7/10. Radiation occasionally occurs into the buttock areas bilaterally and the patient feels the symptoms markedly lessen with activity. Physical examination shows marked forward stooping of the thoracic and cervical spine, with the lower spine showing presence of a substantial reduction in lateral flexion. Question Highlights What is the most likely diagnosis? 1 Spondylolisthesis 2 Osteoarthritis 3 Rheumatoid arthritis 4 Ankylosing spondylitis 5 Psoriatic arthritis
Correct Answer: Ankylosing spondylitis
A 19-year-old man presents with pain and deformity of his right dominant shoulder after a sudden jerking movement to the same shoulder from a wrestling competitor approximately 1 hour ago. He states he felt a clunking sensation when it happened. He was unable to continue wrestling and has pain with movement of the right shoulder. Question Highlights What initial imaging should be the next step? 1 MRI of the shoulder 2 CT of the shoulder 3 Anterior/posterior and internally rotated humeral view radiographs 4 Anterior/posterior, scapular lateral, and axillary radiographs 5 Internal and external radiographs of the humeral head
Correct Answer: Anterior/posterior, scapular lateral, and axillary radiographs The patient's history indicates a potential shoulder dislocation, with an anterior dislocation being more common. The initial imaging recommended to evaluate this diagnosis is radiographs with AP, scapular (Y view), and axillary views. An anterior shoulder dislocation can be seen on the AP view. A scapular (Y view) should be obtained to assess the position of the humeral head. There can be false positives on the scapular view, so axillary view should be performed. An axillary view taken with the arm in abduction is the best way to visualize a posterior dislocation.
A 40-year-old African American woman presents with diffuse headache and joint pain. The headache started few days ago; it is dull and becoming progressively worse. Joint pain is localized in fingers, starts in the morning, and improves during the day; it returns when she gets tired. 2 months ago, she was treated in the ED because of several weeks of lasting fatigue, low-grade fevers, joint pain, hair loss, and oral ulcers. Her laboratory tests were normal, except for positive VDRL and antinuclear antibody tests results. Urine and blood cultures were negative for evidence of infection, and her chest X-ray was normal. Because she felt better after a 10-day tapering course of prednisone, she did not follow up with a healthcare provider at that time. She did not appear to the scheduled control. Today, your examination reveals an ill-appearing woman in distress. Her temperature is 39°C. Her fingers are swollen and red, and she has a malar rash and oral ulcers. Question What additional test should you order as the most specific for her condition? 1 Synovial fluid analysis 2 Urine protein/creatinine ratio 3 Antibodies to the Sm antigen 4 Cerebrospinal fluid analysis 5 Complement system test
Correct Answer: Antibodies to the Sm antigen The diagnosis of SLE is based on clinical features and the presence of autoantibodies. Current criteria for the diagnosis are ≥4 of the following: malar rash discoid rash photosensitivity oral ulcers arthritis serositis renal disorder neurologic disorder immunologic disorder antinuclear antibodies This patient meets the criteria. She has a history of positive ANA and alopecia; there is the presence of a neuropsychiatric disorder, oral ulcers, arthritis, and fever, with infection ruled out. Antibodies to the Sm antigen in the presence of characteristic clinical picture are the most specific for SLE. VDRL is often false positive in SLE patients (in about 1/4 of patients). Clinically, VDRL-positive patients present differently than VDRL negative.
A 32-year-old man is working on his farm when a wild horse suddenly jerks the bridle, forcing the man's right arm over his head and backward. There is a "clunking" sound, and the man is in immediate pain and unable to move his arm. There is a deformity of the shoulder with a depressed area (dimple) noted in the anterior shoulder. Question Highlights What nerve must be carefully assessed in this patient before and after it is fixed because of the possible injury to it? 1 Musculocutaneous 2 Median 3 Axillary 4 Radial 5 Ulnar
Correct Answer: Axillary With a glenohumeral dislocation, as seen in this patient, a risk of neurapraxia of the axillary nerve is possible. Neurovascular function, particularly that of the axillary nerve, must be carefully assessed before and after reduction of the dislocation.
You have inherited three adolescent patients who had different early childhood malignancies that were treated in different ways according to the type and grade of cancer. They have been in remission now for over 5 years. In reviewing their charts, you consider the need to monitor for late effects of childhood cancer. Question Highlights What secondary malignancy is commonly associated with radiation therapy? 1 Bladder cancer 2 Hodgkin's disease 3 Retinoblastoma 4 Bone and soft tissue sarcoma 5 Familial polyposis coli
Correct Answer: Bone and soft tissue sarcoma Radiation is associated with the development of cancer within the radiation field: breast cancer, thyroid cancer, skin cancers, brain tumors, and most commonly bone and soft tissue sarcomas. The cumulative risk for secondary malignancy at age 20 in pediatric cancer survivors is 3-10%, which is 5-20 times greater than for the general population. Survivors should be evaluated yearly for the development of secondary malignancies.
A 40-year-old woman presents occasional constipation that she relates to her diet. She is married with two sons. Aside from mild anxiety, she is dealing appropriately with her duties as a librarian. She does not smoke, she consumes alcohol only recreationally, and she has a body weight of 187 lb. She is well-nourished and communicates well. Lung, cardiac, abdominal, and neurological examinations are unremarkable. Thyroid is normal in size and consistency. Menses are normal, and drug history is negative except for "hormone pills" for contraception. Family history is remarkable for a mother with hypothyroidism and osteoporosis. Question Highlights After reviewing this patient's case history, what imaging modality would you recommend for osteoporosis screening and when should it be done? 1 Bone densitometry after menopause 2 Hip and spinal X-ray after menopause 3 Bone densitometry at age 65 4 Hip and spinal X-ray at age 65 5 Bone densitometry at age 55
Correct Answer: Bone densitometry at age 65 Bone densitometry, or dual-energy X-ray absorptiometry (DEXA scan), from the femoral neck is currently the best screening measure for osteoporosis. The US Preventive Services Task Force (USPSTF, 2002) recommends screening for osteoporosis among women ≥65 and women 60-64 with risk factors for osteoporotic fractures. Risk factors for osteoporosis include history of previous fracture, low body weight, parental history of hip fracture, current smoker, rheumatoid arthritis, increased alcohol consumption, and steroid use. This patient is 40 and does not have additional risk factors to indicate early screening thus the best answer is bone densitometry at age 65.
A 5-year-old African American girl with sickle cell disease presents due to right leg pain. She began to experience right thigh pain and a slight limp following a playground injury 2 weeks ago. Upon sustaining the injury, she was taken to a local ER; X-rays were negative for evidence fracture. She was diagnosed with and treated for a leg contusion. Over the past 2 weeks, the pain has become more severe and she has experienced fever; temperatures have been as high as 102°F, and she has experienced episodic chills. On physical examination, her temperature is 101°F; her right leg is swollen, tender, and erythematous over the anterior aspect of the thigh. The remainder of the PE is unremarkable. CBC with differential reveals a leukocytosis; there is a predominance of neutrophils and a bandemia. Question Highlights What is the most appropriate empiric treatment? 1 Ceftriaxone and vancomycin 2 Gentamicin and vancomycin 3 Nafcillin and a fluoroquinolone 4 Nafcillin and vancomycin 5 Vancomycin and a fluoroquinolone
Correct Answer: Ceftriaxone and vancomycin The correct response is ceftriaxone and vancomycin. This patient most likely has hematogenous osteomyelitis. Osteomyelitis refers to inflammatory destruction of bone due to infection. There are two main categories of osteomyelitis: Hematogenous osteomyelitis, which is most commonly found in children and occurs secondary to sepsis Osteomyelitis directly resulting from the spread of bacteria from an adjacent infection (infected diabetic foot ulcer), trauma (open fractures), or vascular insufficiency (peripheral vascular disease). Osteomyelitis resulting from bacteremia is a disease most frequently associated with sickle cell disease, injection drug users, diabetes mellitus, and the elderly population. Patients with this form of osteomyelitis often present with sudden onset of high fever, chills, pain, and tenderness of the involved bone. Among patients with hemoglobinopathies such as sickle cell disease, osteomyelitis is caused most often by Salmonella paratyphi. Staphylococcus aureus is the second most common cause of osteomyelitis in SCD patients. Any empiric antibiotic regimen will need to provide adequate cover for both of these organisms.
A 66-year-old African American man was recently diagnosed with osteoarthritis of the right knee as evidenced by radiographs. The patient failed conservative treatments with topical NSAIDs and capsaicin and presents for follow-up to further discuss his treatment options. The patient has a known drug allergy to venlafaxine, which resulted in a severe anaphylactic reaction. Current Problem Associated Intervention Uncontrolled type 2 diabetes mellitus metformin 1000 mg twice daily Gastroesophageal reflux disease famotidine 40 mg twice daily Major depressive disorder no active treatment (in remission) Question What oral medication is the best treatment option for this patient? 1 Celecoxib 2 Diclofenac 3 Prednisone 4 Indomethacin 5 Duloxetine
Correct Answer: Celecoxib The best treatment option for this patient is Celecoxib, a selective cyclooxygenase (COX)-2 inhibitor. "The principal benefit with the selective COX-2 inhibitors is the production of comparable analgesia and anti-inflammatory effects to the non-selective non-steroidal anti-inflammatory drugs (NSAIDs), but with fewer symptomatic gastric and duodenal ulcers and a decrease in gastrointestinal symptoms." Further, "in patients with comorbidities such as diabetes, hypertension, and advanced age, a COX-2 selective NSAID or a non-selective NSAID associated with a proton-pump inhibitor should be used."
An 18-year-old man presents 30 minutes after falling on his outstretched arm while skateboarding. He is guarding his left forearm near his wrist with his right hand, and he has his left arm against his body for support. There is a deformity similar to the image with edema and ecchymosis at the wrist. He is neurovascularly intact and the skin is closed. His left elbow and shoulder exam are normal. Question Highlights With this type of deformity, what fracture is most likely? 1 Smith fracture 2 Torus fracture 3 Salter-Harris IV fracture 4 Colles' fracture 5 Die-punch fracture
Correct Answer: Colles' fracture This patient presents with a Colles' fracture. This fracture causes the "dinner fork" appearance of the wrist; the hand and distal fracture fragment of the distal radius (or distal radius and ulna) are dorsally angulated. This injury typically is caused by a FOOSH (fall on outstretched hand) injury.
A 5-year-old girl presents after falling off a shopping cart, tripping, and then falling onto her right arm. On examination, temp is 98.7, pulse 97, respirations 18, blood pressure 127/80 mm Hg. She is alert, oriented, and in no acute distress. Significant findings related to the right arm, which was mildly swollen, deformed, and diffusely tender. There was decreased range of motion of the right elbow due to pain. Sensation was intact. Pulses are within normal limits bilaterally. A radiographic examination was performed. Refer to the image. Question Highlights What is the most common long-term complication associated with the radiology finding? 1 Cubitus varus 2 Myositis ossificans 3 Median nerve injury 4 Ulnar nerve injury 5 Volkmann contracture
Correct Answer: Cubitus varus This patient has a supracondylar fracture. Cubitus varus is an elbow deformity with a decreased carrying angle. There is deviation of the forearm toward the midline of the body. It is the most common complication of a supracondylar fracture of the elbow, usually caused by poor reduction of the fracture with medial displacement, internal rotation, and extension of the distal fragment. The distal fragment then tilts into a varus position. The use of AP, lateral, and oblique view X-ray and CT scan to determine the position of the distal fragment and ensure that its rotation is less than 10° is helpful in preventing this. It produces a cosmetic deformity, but it is usually of little functional significance.
A 47-year-old obese woman presents with left non-dominant shoulder pain and limited motion. The pain began about a month ago but has reduced over time. Her shoulder has progressively lost motion during that time; she cannot reach overhead with that arm. There is no history of trauma, it does not wake her at night, and she can sleep on the left side. Her past medical history is significant for type 1 diabetes and hypothyroidism. She is on regular insulin, NPH insulin, and levothyroxine. Question What finding do you expect on physical exam? 1 Decreased active range of motion, full passive range of motion 2 Decreased active and passive range of motion ≥50% 3 Minimal pain with motion, especially at extremes 4 Weakness with isometric rotator cuff muscle testing 5 Tenderness of the sternoclavicular joint
Correct Answer: Decreased active and passive range of motion ≥50% This patient has adhesive capsulitis (frozen shoulder). Diabetes and hypothyroidism are common risk factors. Pain usually precedes the progressive stiffness. Age is commonly 40-60. Loss of active and passive range of motion by ≥50% is the hallmark of this disorder. Motion is painful, especially at the extremes. Tenderness is common at the deltoid insertion, not over the SC joint. The rotator cuff muscles are not as weak with isometric testing, as if the rotator cuff muscle-tendon were inflamed. This is a disorder of the glenohumeral joint capsule, not the rotator cuff muscles. There may be some weakness due to atrophy from non-use because of the pain and stiffness.
A 43-year-old woman presents with reddish-pink eruptions on her neck, upper back, shoulders, and arms. The skin lesions appear annular and do not hurt or itch. The patient is concerned about having a contagious disease, but she does not feel sick. Her body temperature is 38.4°C, her pulse is 65, and her blood pressure is 150/70. She has a history of hypertension and gastritis, which she has been treating for several years. She exercises regularly, mostly outdoors, and uses sunscreen whenever she feels it is needed. Question Highlights What is the most appropriate next step in the management of this patient? 1 Request anti-Ro/SSA and anti-La/SSB serological tests. 2 Prescribe a topical antibiotic cream. 3 Request her immunization records. 4 Determine the medication history of the patient. 5 Instruct patients about sun-avoidance techniques.
Correct Answer: Determine the medication history of the patient. The lesions described are typical of subacute cutaneous lupus erythematosus (SCLE), a condition that can be induced by certain antihypertensive agents or some proton pump inhibitors used to treat gastritis. The patient has a history of hypertension and gastritis, so you should determine the medication history of the patient.
A 12-year-old boy who presented to the emergency department 3 days ago with a right ankle injury. He "rolled over" his ankle playing football. Radiographs were negative. He has been treating with rest, icing, compression, and elevation (RICE). Past medical history is unremarkable, without prior ankle injuries. Today, he has mild pain and swelling of the outer aspect of the right ankle near the lateral malleolus with mild tenderness on motion, especially inversion, with a mild limitation of motion due to pain. He can almost fully bear weight on the ankle. There is no joint instability. Question What is your next step in managing this injury? 1 MRI of the right ankle 2 Hard casting for 6 weeks 3 Early mobilization 4 Hydrocodone 5 mg 5 Rigid brace
Correct Answer: Early mobilization Ankle sprains are among the most common sports injuries, with inversion injuries resulting in lateral sprains being the most common type. The above represents a grade I sprain since there is no joint instability. After an initial period of rest, icing, compression, and elevation, if the ankle is stable, mobilization with range of motion exercises progressing to ankle strengthening should begin as soon as possible to avoid atrophy and joint stiffness. Protected weight-bearing may occur by using crutches and an ankle stabilizer.
A 43-year-old woman who works as a secretary and spends most of her day typing on the computer has symptoms of carpal tunnel syndrome. Question Highlights What test is most useful in confirming the most likely diagnosis? 1 Electromyogram 2 Computerized tomogram (CT) of the wrist 3 Magnetic resonance image (MRI) of the wrist 4 Plain radiographs of the wrist 5 Erythrocyte sedimentation rate (ESR)
Correct Answer: Electromyogram Electrophysiologic (electromyogram) testing is the most useful confirmatory test for carpal tunnel syndrome (CTS), but these tests must be used with caution. Some patients have no clinical signs or symptoms but will have an abnormal EMG; as many as 5-10% of patients with CTS will have normal results. In general, the diagnosis of CTS is based on your H&P, with EMG use to confirm your suspicion. A positive Phalen's and Tinel's sign would demonstrate paresthesia in a median nerve distribution, a positive Tinel's sign via percussion of the median nerve at the wrist, and positive Phalen's after passive flexion of the hand at the wrist. They are used to help in diagnosis but are not very sensitive or specific tests. Electrodiagnostic studies are often needed to confirm carpal tunnel syndrome.
A 51-year-old woman presents due to menstrual irregularity, hot flashes, and mood changes. Physical examination reveals an atrophic vagina and breasts that have decreased in size. You explain that those symptoms and signs are physiologic and related to her age. The patient decides not to start hormonal replacement therapy (HRT). 8 years later, a dual-energy X-ray absorptiometry (DEXA) reveals a T score of -3. Question Highlights Change in the serum level of what hormone or electrolyte is most likely responsible for the alterations detected by the DEXA scan? 1 Calcium 2 Cholecalciferol 3 Parathyroid hormone 4 Estrogen 5 Progesterone
Correct Answer: Estrogen Estrogen levels fall around and after menopause due to age-linked decline in number of ovarian follicles. This permanent loss of estrogen may result in the increase of bone reabsorption and osteoporosis. Normally, estrogen inhibits apoptosis in osteoblasts (bone-forming cells) and induces apoptosis in osteoclasts (bone-resorbing cells). The absence of this hormone will trigger bone reabsorption, leading to loss of bone mass and interconnections in trabecular bone. This is evidenced the DEXA T score of -3.
A 13-year-old boy presents with pain in his right leg. The pain has been present for about 2 months, but it has worsened with time; he has also developed a low-grade fever. He denies any known injury to the area. On examination, there is tenderness and mild swelling near the right fibula. X-ray reveals an invasive lesion involving the right fibula, with a periosteal onion-skin reaction. Question What is the most likely diagnosis? 1 Plasmacytoma 2 Ewing's sarcoma 3 Multiple myeloma 4 Endochondroma 5 Osteochondroma
Correct Answer: Ewing's sarcoma Ewing's sarcoma is a neoplastic tumor of the bone that most commonly affects the lower extremities, including the long bones and pelvic bones. Patients typically present with swelling and pain. Classic X-ray findings show a destructive lesion described as "moth eaten" or periosteal onion-skin reaction.
A 42-year-old woman presents with a history of chronic fatigue and pain around her neck, shoulders, and lower back. She is also experiencing chronic headaches and irritable bowel symptoms. Upon physical exam, no abnormal findings were found except for trigger points that produced pain around the trapezius, lateral epicondyle of her elbow, and the medial fat pad of her knee. Laboratory findings showed a normal ESR, negative RF factor, and a negative ANA. Question Highlights What is the most likely diagnosis? 1 Fibromyalgia 2 Osteoporosis 3 Polymyalgia rheumatica 4 Rheumatoid arthritis 5 Systemic lupus erythematous
Correct Answer: Fibromyalgia This patient presents with symptoms that are found in patients with fibromyalgia, such as common trigger points noted in her physical exam. Lab findings help rule out other options, as patients with fibromyalgia have normal lab findings.
A 19-year-old man presents with pain and deformity of his right dominant shoulder after a sudden jerking movement to that shoulder from a wrestling competitor approximately 1 hour ago. He was unable to continue wrestling and has pain with any movement of the right shoulder. On exam, you see a loss of normal shoulder contour anteriorly. There is no focal joint or bony tenderness. Question Highlights What is the most likely diagnosis? 1 Rotator cuff tear 2 Shoulder separation 3 Glenohumeral dislocation 4 Fractured clavicle 5 Sternoclavicular subluxation
Correct Answer: Glenohumeral dislocation With an acute shoulder or glenohumeral dislocation, there is considerable pain with any movement. There is more loss of normal shoulder contour with anterior dislocation than with posterior dislocation. With a rotator cuff tear, there is gross weakness to resistance and gravity. Shoulder (acromioclavicular) separation, clavicle fracture, and sternoclavicular subluxation would have tenderness and pain directly over the injury site.
A 52-year-old African American man presents with symptoms of redness, swelling, and pain in his left great toe. He states that the symptoms started at approximately 3 AM and have persisted since onset. He admits one previous episode over the past month but notes resolution of his symptoms without any treatment. His only current medication is chlorthalidone for essential hypertension. On exam, you note a moderately obese male with edema, erythema, and exquisite tenderness of the left metatarsophalangeal joint. There are no obvious deformities or evidence of trauma. Vital signs are within normal limits. Question What is the most likely diagnosis? 1 Cellulitis 2 Gout 3 Hallux valgus 4 Rheumatoid arthritis 5 Stress fracture
Correct Answer: Gout Gout typically presents with redness, swelling, and exquisite tenderness of a single joint, commonly the first MTP (metatarsophalangeal) joint or "great toe" (podagra). Flares commonly present at night (between midnight and 8 AM) and may resolve within days to weeks early in the disease, even without treatment. Thiazides (e.g., chlorthalidone) commonly incite gout attacks due to impaired elimination of uric acid by the kidneys. Alcohol consumption is similarly known to trigger gout attacks.
This baby is at risk for hemorrhagic disease of the newborn because he was born at home and may not have had immediate and routine medical care. A decrease in factors II, VII, IX, and X normally occurs in all newborns by 48-72 hours. There is a gradual return to birth levels by 7-10 days of life. Routinely in a hospital setting, an intramuscular dose of vitamin K immediately after birth helps prevent hemorrhagic disease of the newborn. The disease is treated with an intravenous infusion of vitamin K, with improvement in coagulation defects within a few hours. Pressure and chemocautery will not correct the coagulation factors. Plasma and blood transfusions may be needed if the baby has a severe form of vitamin K deficiency and his PT fails to improve, or if the bleed is severe. Vitamin C administration plays no direct part in the immediate treatment of this scenario. 1 Closure of the wound 2 Debridement 3 Hemostasis 4 History of wound mechanism 5 Tetanus immunization
Correct Answer: Hemostasis Hemostasis is foremost in the evaluation of a wound in any setting. Large amounts of blood pooling or notable pulsations warrant urgent exploration for arterial damage, which could need emergent clamping and ligation.
A 63-year-old man with long-standing right shoulder pain from impingement syndrome presents because he cannot lift his arm after pulling the starter cord on his lawn mower 1 month ago. An AP view of his right shoulder is ordered. Question What finding on the plain films may indicate rotator cuff arthropathy? 1 A >2 cm widening of the acromioclavicular joint 2 Cystic changes in the head of the humerus 3 Narrowed glenohumeral joint space 4 High-riding humeral head 5 A Bankart lesion of the glenoid
Correct Answer: High-riding humeral head With large long-standing tears of the rotator cuff, AP radiographs may reveal a high-riding humerus relative to the glenoid (subluxed), indicative of rotator cuff arthropathy.
A 34-year-old man is brought to the emergency department after a single-car motor vehicle collision. Preliminary radiologic studies show a comminuted fracture of the left tibia. The patient is describing a markedly increasing amount of pain felt in the injured extremity. He describes the pain as being a 10/10 that is becoming worse with each passing minute. With anguish on his face, he describes it as a deep achy burning pain. You quickly examine the left leg and note pallor, a tense "wood-like" feeling of the extremity, diminished sensation, and muscle weakness. Question Highlights Based on the most likely diagnosis, what is the best pharmaceutical intervention for this patient's pain? 1 IV opioids 2 Oral acetaminophen 3 Oral NSAIDs 4 PCA opioids 5 Topical NSAIDs
Correct Answer: IV opioids This patient is experiencing acute compartment syndrome (ACS), a medical emergency that can have significant consequences. ACS most often develops after significant trauma, especially involving a long bone fracture. Fractures account for more than 75% of cases of ACS, even higher with comminuted fractures. Signs and symptoms of ACS often occur in a stepwise fashion; an important clue is a rapid progression of symptoms and signs in a short time. Symptoms include pain out of proportion to the apparent injury, persistent deep ache or burning pain, or paresthesias. Signs include pain with passive stretch of muscles in the affected compartment, a tense compartment with a firm "wood-like" feeling, pallor, diminished sensation, muscle weakness, and a late finding of paralysis. Adequate decompressive fasciotomy must be performed as soon as possible. Due to the severity of pain, the most appropriate answer for pain management would be IV opioids. Patient-controlled analgesia (PCA) opioid route may be considered a viable option, but not until intervention has occurred; it could mask the severity of the pain and cause complications. Due to the intense and severe pain that this condition creates, oral acetaminophen, oral NSAIDs, and topical NSAIDs are unlikely to be effective.
A 47-year-old woman with a history of obesity presents to the emergency department due to pain in her right ankle after a fall 2 days ago. Upon physical exam, she has pain and tenderness at the tip of the medial malleolus and has the inability to bear weight for at least 4 steps. The patient does not present with pruritus or any rashes, but the ankle appears swollen. The patient's foot appears to be neurologically intact. She does not present with a fever or erythema of the ankle. According to the Ottawa Ankle Rule, she qualifies for X-rays. Question Highlights What first-line treatment can be used to help treat this condition? 1 NSAID 2 Immobilization 3 Opioid 4 Ambulation 5 Antibiotic
Correct Answer: Immobilization Immobilization in a temporary cast/splint is the treatment for a stable fracture that requires temporary immobility, rest, ice, and elevation. Increasing ambulation on the affected ankle would not be recommended.
A 17-year-old male college freshman presents with fatigue, back pain, and stiffness. The pain has been present for the past several months, but it appears to be worsening. The back symptoms are worse at night and first thing in the morning; they improve somewhat during the day. He reports the pain improves with exercise. He is having difficulty staying productive at school because he is always tired. Question Highlights What laboratory finding is most probable? 1 Decreased c-reactive protein 2 Increased b-natriuretic protein 3 Increased erythrocyte sedimentation rate 4 Decreased alkaline phosphatase 5 Increased uric acid level
Correct Answer: Increased erythrocyte sedimentation rate This patient presents with signs and symptoms indicative of inflammatory arthritis, most likely ankylosing spondylitis (AS). Characteristic symptoms include back pain that is worst in the morning and improves with exercise. Back stiffness and extreme fatigue are also reported. AS is more common in male patients. As with other inflammatory arthritides, patients usually present at a younger age (18-35) relative to patients with mechanical causes of back pain or arthritis. AS is a clinical diagnosis, but many patients will have elevated erythrocyte sedimentation rates and c-reactive protein rates, both of which are non-specific markers of inflammation. Approximately half of patients will have an increased alkaline phosphatase level. HLA-B27 test may also be completed in the workup.
A 27-year-old Caucasian man returns to the emergency department with unbearable left lower leg pain approximately 6 hours after initial discharge. While playing lacrosse, the patient sustained a closed mid-shaft tibial fracture. After casting and an anti-inflammatory, his pain was noted to be mild (2/10) at time of discharge. He reports his pain is increasing dramatically (it is now rated 9/10) and is unresponsive to his prescribed narcotic, acetaminophen, icing, and elevating his leg. He also describes a feeling of tingling and numbness throughout the lower left extremity. The patient arrived on crutches and appears in obvious pain. He is afebrile; he has a pulse of 105. The cast is intact and the remainder of the left leg and foot is examined. The patient is tender to palpation of the left foot and ankle, with swollen firm tissue. The skin appears shiny. Pulses are normal, but capillary refill and sensation are decreased on the left foot compared to the right. With passive movement of the patient's left toes, he cries out in pain. There is no ecchymosis or visible skin lesions on the left foot or ankle. Question Highlights What explains the physiologic basis for this patient's severe pain? 1 Chronic peripheral vascular disease, leading to poor arterial blood flow 2 Demyelination of the neuron sheath, leading to pain and paresthesias 3 Immobility of the affected limb, leading to venous stasis and thrombosis 4 Increased dopamine release, leading to activation of the reward centers 5 Increased pressure within the tissue space, leading to venous obstruction
Correct Answer: Increased pressure within the tissue space, leading to venous obstruction This patient is exhibiting a presentation for compartment syndrome, the basic process of which is increased pressure within the tissue space, leading to venous obstruction. As the pressure rises, muscle and nerve necrosis may occur. Compartment syndrome is an emergent condition that may arise after significant trauma, such as a fracture or burn. It must be quickly identified and treated to prevent irreversible tissue damage.
A 40-year-old man presents with ongoing back pain; there is increased stiffness in the morning, and the pain has been going on for a few months. The patient reports that it can take him up to 30 minutes after waking up for the discomfort to improve. Physical examination and diagnostic testing confirms the diagnosis of ankylosing spondylitis. Question In addition to regular exercise, what medication will best control the patient's symptoms? 1 Acetaminophen 2 Indomethacin 3 Prednisone 4 Aspirin 5 Oxycodone
Correct Answer: Indomethacin First-line treatment of symptomatic ankylosing spondylitis are NSAIDs such as indomethacin. These are effective in controlling the symptoms of ankylosing spondylitis and beneficial due to their anti-inflammatory properties.
A 13-year-old girl presents for her school physical. On examination, you notice the posterior curvature of her thoracic spine to be very prominent and bulging backward. Question Highlights What type of deformity of the spine does she have? 1 Kyphosis 2 Scoliokyphosis 3 Scoliosis 4 Lordosis 5 Gibbus
Correct Answer: Kyphosis Kyphosis is a posterior convex angulation of the thoracic spine as evaluated on a side view. Excessive and exaggerated angulation results in cosmetic problems, back pain, and cardiorespiratory problems. Kyphosis may be the post-traumatic or non-traumatic type. Scheuermann's kyphosis and postural, congenital, and metabolic bone disease are a few of the non-traumatic causes of kyphosis.
A 67-year-old man presents with the complaint of a "tender bulge in his right dominant arm" following an attempt at lifting his couch 2 days ago. He admits to having mild chronic shoulder pain and taking ibuprofen (Motrin) for relief. While attempting to lift the couch, he felt a snap in his shoulder and noticed a bulging in his anterior arm. Question What anatomic structure is most likely injured? 1 Distal biceps brachii tendon rupture 2 Long head of the biceps brachii tendon rupture 3 Short head of the biceps brachii tendon rupture 4 Supraspinatus tendon rupture 5 Triceps brachii tendon rupture
Correct Answer: Long head of the biceps brachii tendon rupture The long head of the biceps brachii passes through the bicipital groove of the humeral head and attaches to the superior rim of the glenoid. It functions to suppress the humeral head on the glenoid surface, but it is exposed to friction rubbing over time. Therefore, it is subject to rupture, even with what seems like minimal trauma.
A 16-year-old boy presents following the striking of a wooden door with a closed fist an hour ago when he was angry at his mother. He is neurovascularly intact, and the skin is closed. There is an obvious deformity with a loss of small finger metacarpal knuckle. Radiographs reveal an oblique mid-shaft fracture of the fifth metacarpal with a palmar angulation of 45°. Question Highlights What physical exam technique must be performed to check for rotation of the fracture? 1 Look at the fingernails while fingers are extended for position in relationship with each other. 2 Look for finger malposition when the fingers are flexed into the palm. 3 Look for folds in the skin over the metacarpal head. 4 Check for motion of the PIP and DIP joints. 5 Look for palmar edema of the affected digit metacarpal.
Correct Answer: Look for finger malposition when the fingers are flexed into the palm. With a fractured fifth metacarpal, there is an increased incidence of malrotation of the distal digit due to the potential loss of metacarpal height and lack of ligamentous support of the metacarpal head by the intermetacarpal ligaments. Therefore, rotation must be checked closely to ensure proper position of the distal fracture segment in relation to the proximal fracture fragment for functional healing to take place. This is best accomplished by asking the patient to make a partial fist (gently) and watching for finger malposition (normally all the digits of a closed fist point towards a single spot on the scaphoid) or "cross-over." Be sure to compare the injured hand to the uninjured one as some "normal cross-over" can be seen with the fifth metacarpal.
A 19-year-old man presents with pain of his right dominant shoulder after a sudden jerking movement to the same shoulder from a wrestling competitor approximately 1 hour ago. He states he felt a clunking sensation when it happened. He was unable to continue wrestling and has pain with movement of the right shoulder. Question Highlights What finding do you expect on physical exam? 1 Tenderness to palpation directly over acromioclavicular joint 2 Loss of normal surface contour of the shoulder 3 Full passive range of motion 4 Ecchymosis over the clavicle extending to the shoulder 5 Full active motion of the shoulder
Correct Answer: Loss of normal surface contour of the shoulder The patient's history is consistent with a potential shoulder dislocation. With the dislocation of the glenohumeral joint, there will be a loss of normal contour of the shoulder's surface anatomy.
A 75-year-old man presents to his pain management office for follow-up after a lumbar laminectomy 2 weeks ago. He states that he has had a fever for the past 6 days. He denies any headache, chest pain, shortness of breath, cough, abdominal pain, nausea, diarrhea, vomiting, or extremity pain. He has a past medical history of hypertension, hyperlipidemia, chronic lumbar stenosis (now status post laminectomy), and type 2 diabetes mellitus. He also has a history of intravenous drug use. He was referred for an MRI of the spine, which revealed destruction of the vertebral body of L4 with significant collapse. Question Highlights What is the most likely additional clinical manifestation in this patient? 1 Foul urinary odor 2 Lower back pain 3 Lower extremity sensory loss 4 Presence of a sinus tract 5 Bowel incontinence
Correct Answer: Lower back pain This patient is experiencing acute osteomyelitis secondary to the recent lumbar laminectomy. His risk factors include his recent surgery, history of IV drug use, and diabetes mellitus. Most cases are monomicrobial in etiology. Of episodes of acute vertebral osteomyelitis, 40-50% are caused by Staphylococcus aureus. The signs and symptoms of vertebral osteomyelitis are non-specific. Only about 50% of patients develop fever >38°C (100.4°F), perhaps because analgesic drugs are frequently used by these patients. Lower back pain is the leading initial symptom (>85% of cases). The location of the pain corresponds to the site of infection: the cervical spine in ~10% of cases, the thoracic spine in 30%, and the lumbar spine in 60%. Local pain, swelling, erythema, and edema, scars, local disturbance of wound healing, and the cardinal signs of inflammation may all be observed on physical exam. If periosteal tissues are involved, point tenderness may be present.
A 73-year-old man presents with the inability to actively raise his left non-dominant arm to reach his kitchen cabinet. This began a month ago after his shoulder pain improved. He had a history of pain in that shoulder for over 6 months that kept him from sleeping on the left side and the pain would often wake him. There was no specific injury he can recall, but he felt a pop a month ago while taking out the trash. Question Highlights What is the diagnostic study of choice if surgery is indicated? 1 CT without contrast 2 Electromyelogram 3 Ultrasound 4 MRI 5 Plain films
Correct Answer: MRI This older man likely has a torn rotator cuff. If surgical treatment is being considered, MRI is the imaging study of choice because it can provide additional information on the status of the muscle and the size and location of the rotator cuff tear.
A 47-year-old obese woman presents with left non-dominant shoulder pain and limited motion. The pain began about a month ago but has reduced over time. Her shoulder has progressively lost motion during that time; she cannot reach overhead with that arm. There is no history of trauma, it does not wake her at night, and she can sleep on the left side. Her past medical history is significant for type 1 diabetes and hypothyroidism. She is on regular insulin, NPH insulin, and levothyroxine. What diagnostic study would best help evaluate the suspected diagnosis? 1 Plain radiographs 2 CT 3 EMG 4 Ultrasound 5 MRI
Correct Answer: MRI This patient presents with adhesive capsulitis (frozen shoulder). Adhesive capsulitis is typically a clinical diagnosis. Imaging is ordered if there is a delay in recovery. MRI can substantiate a frozen shoulder by demonstrating coracohumeral ligament thickening and axillary joint capsule enhancement. MRI is also useful to rule out a possible rotator cuff tear or intra-articular pathology if the patient is not improving.
You are evaluating a 34-year-old African American man for a 5-week history of increasing right groin pain. He denies any injury or history of similar pain. The pain is worse with movement and has progressed to the point that the patient has severe pain with bearing weight. He denies fever, chills, urinary symptoms, or any other issue at this time. He has a past medical history of sickle cell disease and hypothyroidism. Physical exam reveals tenderness upon palpation of the groin with increased pain on both active and passive range of motion of the hip. Homan's sign is negative. Distal pulses and sensation are intact and normal. Question Highlights What diagnostics study should be ordered at this time? 1 Ultrasound of the right lower leg 2 CBC 3 MRI of the right hip with and without contrast 4 CXR 5 Urinalysis
Correct Answer: MRI of the right hip with and without contrast This patient's presentation is concerning for avascular necrosis of the right hip. The most appropriate diagnostic test to order at this time is an MRI of the hip with and without contrast. Avascular necrosis (osteonecrosis) is a focal infarct of the bone due to an interruption of blood supply. Avascular necrosis (AVN) most commonly occurs in the hips, jaw, and scaphoid bone of the hand. Risk factors for AVN include trauma, coagulopathy, alcoholism, chronic corticosteroid use, sickle cell disease, and autoimmune diseases such as RA and SLE.
A 27-year-old man presents with knee pain that started while he was playing basketball the previous afternoon. The patient states that he twisted his left knee with his left foot still planted on the floor as he attempted to pass the ball to a teammate. He describes a popping sensation at the time of the injury, but he could still bear weight on his leg. He does not recall any immediate swelling. The morning of presentation, the patient noted swelling of the knee and pain that was significantly worse than it was the day before. He has taken 600 mg ibuprofen for the pain but has experienced only mild relief. Upon examination of the left knee, there is a limitation in range of motion, the presence of medial joint line tenderness, and an effusion. McMurray's test is positive. Question Highlights What is the optimal imaging modality or procedure for confirming the suspected diagnosis? 1 Anteroposterior (AP) and lateral X-ray 2 Ultrasonography 3 Computed tomography (CT) scan 4 Magnetic resonance imaging (MRI) 5 Positron-emission tomography (PET) scan
Correct Answer: Magnetic resonance imaging (MRI) This patient most likely has a medial meniscal injury of the left knee. Meniscal tears are the most common knee injuries encountered in primary care. They result from a twisting action exerted on the knee joint while the foot is still in a weight-bearing position. Magnetic resonance imaging (MRI) is still the optimal test for confirming the diagnosis of a meniscal injury. All intra-articular structures (menisci, ligaments, and articulating surfaces) can be visualized in great detail.
A 50-year-old obese woman presents with severe left knee pain. She states the pain began about 8 months ago but has gotten significantly worse in the last 3 months. The patient denies any trauma or event that initiated the pain. She notes stiffness in the knee first thing in the morning; it only lasts around 5-10 minutes. The knee pain worsens with activity and is relieved with rest. The patient's medication list includes lisinopril 10 mg once daily for high blood pressure. She has a documented medication allergy to acetaminophen, which gives her hives. Physical examination reveals a female with a BMI of 40, limited range of motion of the left knee, and crepitus. Question Highlights Considering the most likely diagnosis for this patient, what pharmaceutical regimen would be recommended? 1 Meloxicam 2 Allopurinol 3 Oxycodone 4 Alendronate 5 Risedronate
Correct Answer: Meloxicam This patient most likely has degenerative joint disease, also known as osteoarthritis (OA). OA is the most common form of knee arthritis. Common characteristics of OA include onset after age 40 (usually over 55), obesity, and (typically) genetic predisposition. Patients with OA initially have an insidious onset of pain that will rapidly progress. Pain is exacerbated by activity and relieved at rest. Morning stiffness is common in patients with OA, but it will resolve within 30 minutes. Common symptoms experienced by patients with OA include buckling or giving way of the affected joint due to the bony areas impinging upon each other. Physical examination findings include limited range of motion of the affected joint and crepitus; this patient exhibits both findings. Patients may also have tenderness to palpation of the joint, joint effusion, or even palpable osteophytes. Initially, pharmaceutical management will be to initiate acetaminophen, up to 4 g per day. Since our patient has an acetaminophen allergy, this would not be an appropriate choice. NSAID therapy or a COX-2 inhibitor would be an appropriate next option, making meloxicam the correct answer in this scenario.
A 33-year-old woman presents for routine follow-up with an abnormal computed tomography (CT). This was done due to gradual worsening exertional dyspnea. Her past medical history includes a leg amputation 4 years ago for osteogenic sarcoma treated with neoadjunctive chemotherapy. She is married and a lifelong non-smoker. Her physical exam includes a BP 111/67 mm Hg, pulse 70/min, respirations 14/minute, temperature of 98.4°F. She is in no acute distress. Pulmonary exam reveals clear breath sounds bilaterally. There is a healed laparotomy incision for a colon resection from perforated diverticulitis several years ago. A chest CT demonstrates a new solitary irregular non-calcified 3 cm nodule. What is the most likely diagnosis of this nodule? 1 Primary lung cancer 2 Metastatic osteogenic sarcoma 3 Lymphoma 4 Pneumonia 5 Pulmonary thymoma
Correct Answer: Metastatic osteogenic sarcoma This patient most likely presents with metastatic osteogenic sarcoma. Pulmonary metastases may present with symptoms similar to primary lung cancer. These include chest pain, cough, hemoptysis, and dyspnea. Usually, metastatic lesions are detected with chest X-ray or CT. CT can characterize suspicious findings on chest X-ray and detail the number of metastases. CT detects approximately 80% of pulmonary metastases detected at surgical exploration. Positron emission tomography (PET) scanning is currently being investigated as a modality to access for pulmonary metastases. In patients with a previous history of sarcoma, a new pulmonary nodule has >90% chance of being a metastasis. The lungs are the most common site of metastasis for osteosarcoma.
A 65-year-old woman presents with fatigue, loss of energy, lack of appetite, low-grade fever, muscle and joint aches, and stiffness. She has a glucose-6-phosphate dehydrogenase (G6PD) deficiency and a history of coronary artery disease. On examination, you find swollen, painful, and tender small joints in a symmetrical pattern. Using the history, physical, and X-rays, as well as the presence of rheumatoid factor and antibodies to cyclic citrullinated peptides, you conclude that she has a flare-up of a chronic diagnosis. Question Highlights What long-term therapy should be recommended in order to modify the course of the disease? 1 NSAIDs 2 Methotrexate 3 Sulfasalazine 4 Corticosteroids 5 Selective COX-2 inhibitors
Correct Answer: Methotrexate Methotrexate is usually the initial synthetic disease-modifying antirheumatic drugs (DMARD) of choice for patients with rheumatoid arthritis. It is generally well-tolerated and often produces a beneficial effect in 2-6 weeks.
A 35-year-old man presents with right shoulder pain that is becoming progressively worse. He expresses concern that, although he visits the gym 3 times a week, over the past month he has not been able to increase the amount of weight he lifts secondary to the shoulder pain. He has not tried anything to alleviate the pain. The pain is at its worst at night while he is trying to sleep. He also reports pain while in the shower washing his hair or using the shoulder press machine at the gym. He denies any history of recent trauma or sports-related injury; however, upon questioning, he reports that about 1 month ago he and his wife painted their entire house in a weekend. Upon physical exam of the shoulders, no swelling, atrophy, redness, or bruising is noted. Point tenderness is noted over the right lateral deltoid muscle. Active ROM of the right shoulder at 80 degrees of abduction elicits pain. Patient has a negative drop arm test, negative apprehensive test, and a positive Neer impingement test of the right shoulder. Question Highlights What is the suggested first line of treatment? 1 Rest with arm in sling 2 Modification of activity, NSAIDs, and physical therapy 3 Corticosteroid injections 4 Surgical treatment and corticosteroid injections 5 Opioid analgesics
Correct Answer: Modification of activity, NSAIDs, and physical therapy The first-choice treatment is modification of activities that produce pain, use of nonsteroidal anti-inflammatory drugs (medications may help relieve symptoms), and physical therapy. The goal is pain control and inflammation reduction. Movement is needed to maintain flexibility and range of motion.
A 25-year-old man presents with back pain and stiffness. He states he has had longstanding issues with back pain. He denies any trauma to his back. He has noticed associated increasing stiffness and general fatigue. He feels that these issues have gradually worsened over the last several months and are more persistent recently. He notes that the pain is much worse first thing in the morning, rating it a 6-7/10. Radiation occasionally occurs into the buttock areas and the patient feels the symptoms actually lessen with activity. Physical examination shows marked forward stooping of the thoracic and cervical spine with the lower spine showing the presence of a substantial reduction in lateral flexion. Question Highlights What would be the first-line pharmaceutical treatment to consider in this patient? 1 TNF inhibitors 2 NSAIDs 3 Sulfasalazine 4 Etanercept 5 Corticosteroids
Correct Answer: NSAIDs The correct response is NSAIDs. The patient history and physical examination described above is most likely a case of ankylosing spondylitis (AS). This is a chronic inflammatory disease that consists of many signs of symptoms, specifically significant back pain and progressive spinal stiffness. Many times, patients with AS will also have transient or persistent peripheral arthritis, as well as other manifestations such as anterior uveitis. Typically this is seen in male patients ages 20-30, although some may start having issues as early as their late teens. The goals of management of AS is to maximize the long-term health-related quality of life: relief of symptoms, maintenance of function, prevention of complications from spinal disease, and the minimization of extra-spinal and extra-articular manifestations and comorbidities. The first-line treatment for AS is nonsteroidal anti-inflammatory drugs (NSAIDs). Usually, empiric trials of different NSAIDs are warranted to gain satisfactory and beneficial relief of symptoms.
A 52-year-old overweight woman has had pain in her right hand for the past month. She is employed as a pastry chef and has trouble making a fist. On exam, she is tender over the radial styloid. You have her flex her thumb into her palm and move the wrist into ulnar deviation. This movement recreates her pain. Question Highlights What is the initial treatment for this condition? 1 NSAIDs and thumb spica splint 2 Corticosteroid injection into the tendon sheath 3 Surgical release of the tendon 4 Short arm casting for 6 weeks 5 Joint replacement surgery
Correct Answer: NSAIDs and thumb spica splint DeQuervain's disease is a type of tenosynovitis. The initial treatment is conservative, involving NSAIDs and splinting.
A 50-year-old man was playing baseball with his company team last weekend and is now experiencing severe pain in his left shoulder. He states that he has been the team pitcher for several years now. He has developed pain progressively in his left shoulder over the last few months. He denies any numbness or tingling in his arm, hand, or fingers of his left upper extremity. Question Highlights Which exam would you expect to yield pain? 1 Neer test 2 Tinel sign 3 Phalen test 4 Bragard stretch test 5 Apley grind test
Correct Answer: Neer test Neer test is the only exam listed that evaluates for a rotator cuff injury. The Neer test is performed by having the patient internally rotate the shoulder while the practitioner flexes the arm forward. Pain will be elicited as the supraspinatus muscle presses against the acromion.
A 54-year-old man presents with acute onset of excruciating pain in his right toe. The patient states the pain began shortly after dinner, and it has progressively worsened since then to the point where he is now unable to bear weight on the affected side. In addition, he explains that aside from occasional backaches, he has never experienced pain like this before. He reports overall good health and aside from a multivitamin, he uses no medications or supplements. On exam, the patient's right foot is swollen, and the joint of the great toe is tense and inflamed. His temperature is 37°C, blood pressure is 155/85 mm Hg, and pulse is 103 beats per minute. Labs reveal an elevated serum uric acid level. Question What is examination of synovial fluid from the affected joint space most likely to reveal? 1 Negatively birefringent needle-shaped crystals 2 Positively birefringent needle-shaped crystals 3 Brownish clumps with elevated WBCs and negative bacteria 4 Increased polymorphonuclear leukocytes, WBCs, and positive bacteria 5 Decreased polymorphonuclear leukocytes, WBCs, and negative bacteria
Correct Answer: Negatively birefringent needle-shaped crystals
A 32-year-old man presents with pain in his back, buttocks, and posterior thighs for 2 days after lifting a heavy load at work. He denies pain when sleeping unless he rolls over in bed. He also has pain with just standing or sitting still in a chair. He denies any radicular symptoms or bladder or bowel dysfunction. Question Highlights What diagnostic test does this patient require? 1 MRI of the LS spine 2 CT of the LS spine 3 Plain radiographs of the LS spine 4 Electromyogram 5 No diagnostic tests required
Correct Answer: No diagnostic tests required Most patients presenting with low back pain less than 4 weeks without evidence of neurologic compression do not require any diagnostic tests. Current recommendations by the American College of Physicians advise against routine imaging in patients presenting with low back pain unless neurologic deficits are present. This patient denies any nocturnal symptoms, radicular symptoms, bladder or bowel dysfunction, so imaging is not indicated.
A 63-year-old man presents with pain in both knee joints that began almost a year ago. It was mild and present only during activity, but it has worsened now and is present even during rest. There is no joint stiffness. He has diabetes and has been on insulin treatment for the past 10 years. His BMI is 30. Vitals are within normal limits. Examination reveals pale conjunctivae. On examination of the knee joints, there is no tenderness, warmth, or erythema. Pain in the left knee joint is reproduced on walking. You suspect osteoarthritis and order lab investigations and X-ray. Meanwhile, you prescribe exercises and ibuprofen. What is a risk factor for osteoarthritis in this patient's case? 1 Sex 2 Anema 3 Diabetes 4 Insulin 5 Obesity
Correct Answer: Obesity Obesity is a risk factor for osteoarthritis (OA). OA is a degenerative joint disease commonly seen in advanced age. It is characterized by deep achy joint pain involving the hip, knee, ankle, and foot joints with an insidious onset. The pain is usually present during activity initially and gradually worsens. The gait may be antalgic if weight-bearing joints are involved. The underlying pathology is the breakdown of the articular cartilage in the joint synovial tissue. It is believed to be due to excessive wear and tear, but non-specific inflammatory changes may also occur. Some of the risk factors for OA include: Advanced age Obesity (due to increased weight-bearing stress on the joints). The knee joints are most susceptible. Menopause Trauma Excessive use Infection Certain hemoglobinopathies, previous rheumatoid arthritis, acromegaly, etc.
A 28-year-old man presents due to a left ankle sprain. The patient describes walking in the woods in flip-flops yesterday, inadvertently stepping into a hidden hole. The patient immediately noticed extreme pain with weight bearing on the left ankle, making it extremely difficult for him to walk out of the woods. Ambulation is extremely difficult. He describes the pain as an 8-9/10. The patient admits to previous ankle sprains, but this one has been much more severe and debilitating. He has noted significant swelling and exquisite tenderness to touch. He has treated his sprain with elevation, ice for 20 minutes at a time, and acetaminophen 1000 mg every 6 hours. Physical examination reveals a moderate degree of ecchymosis of the left ankle with substantial mechanical instability and moderate restriction of range of motion. Question Highlights In addition to RICE guidelines and immobilization of the ankle, what pharmaceutical intervention should be initiated to further help reduce swelling and pain? 1 IV opioids 2 Oral NSAIDs 3 PCA opioids 4 Topical lidocaine 5 Topical NSAIDs
Correct Answer: Oral NSAIDs This patient has a grade II ankle sprain, which typically involves an incomplete tear of a ligament. Patients will experience moderate pain, swelling, tenderness, and ecchymosis. There will be mild to moderate joint instability during exam, some restriction of the range of motion, and loss of function. Ambulation and weight bearing are painful. Treatment should follow the MICE mnemonic: modified activities, ice, compression, and elevation. Since this is a grade II ankle sprain, there should be protected weight bearing with crutches and an ankle stabilizer brace. Early motion is essential in this type of sprain; patients should be strongly encouraged to perform home exercise or see a physical therapist. Controlling pain and reducing the swelling are common concerns for most patients. Commonly prescribed medications are oral form analgesics, such as NSAIDs.
A 72-year-old woman with a past medical history of COPD, deep venous thrombosis, breast cancer in remission, hyperthyroidism, and premature ovarian failure presents with sudden-onset severe lower back pain of 2 hours' duration. She states that the pain began when stepping onto the floor as she got out of bed. She denies any falls, prior injuries, genitourinary complaints, or lower extremity numbness, tingling, paresthesias, or weakness. She takes a daily prednisone tablet for COPD. Her social history is notable for a sedentary lifestyle and a 40 pack-year smoking history. Her physical exam reveals a tall thin-framed woman with noticeable lid lag. Her thoracic vertebral body is exquisitely tender at T12, and she has limited mobility of all thoracolumbar planes of motion. Lateral and AP lumbar spine X-ray reveals osteopenia and collapse of vertebral body of T12. Question What is the best next step in managing this patient? 1 Increase the dose of prednisone to reduce symptomatology. 2 Order human osteocalcin and bone alkaline phosphatase levels. 3 Perform a quantitative computerized tomography of the spine. 4 Prescribe estrogen replacement therapy. 5 Order a dual-energy X-ray absorptiometry test.
Correct Answer: Order a dual-energy X-ray absorptiometry test. This patient's symptoms, coupled with the radiographic images, suggest a diagnosis of an osteoporotic vertebral compression fracture of the T12 vertebral body. Risk factors for primary osteoporosis include early menopause or premenopausal estrogen deficiency states, prolonged periods of inadequate calcium intake, sedentary lifestyle, and tobacco and alcohol abuse. Secondary osteoporosis results from chronic conditions that contribute significantly to accelerated bone loss: endogenous and exogenous thyroxine excess, hyperparathyroidism, cancer, gastrointestinal diseases, medications such as corticosteroids, renal failure, and connective tissue diseases). The most widely used techniques for assessing bone mineral density (BMD) are dual-energy X-ray absorptiometry (DEXA) and quantitative computerized tomography (CT). Quantitative CT is more sensitive, but it results in substantially greater radiation exposure than DEXA; for this reason, DEXA is the diagnostic measure of choice. Vertebral fracture assessment is now available through imaging completed with the DEXA assessment and provides a more accurate assessment of the patient's bone density.
A 35-year-old man with no significant past medical history has been experiencing progressive, moderately severe right knee pain for the past 3 months. Pain was initially felt only at night, but it is now constant throughout the course of the day for the last several weeks. It is especially severe upon ambulation and during knee ranges of motion, causing him to limp. He denies fever, chills, weight changes, history of gout, sexually transmitted diseases, hip or back pain, recent instrumentation, trauma, or injuries. His physical exam reveals an antalgic gait with limp, limited ranges of motion of the right knee, and a 3 cm diameter firm, tender mass at the distal femur. There is no erythema, crepitus, alignment deformity, or effusion noted. Question Highlights What is the next appropriate step in the management of this patient? 1 Order a serum uric acid level and prescribe indomethacin. 2 Wrap the right knee in an elastic bandage and prescribe physical therapy. 3 Perform a joint aspiration with microscopic cellular assessment. 4 Obtain a blood culture and begin empiric dicloxacillin. 5 Order an LDH, ALP, and a plain radiograph of the knee.
Correct Answer: Order an LDH, ALP, and a plain radiograph of the knee. This patient's manifestations are most suggestive of osteosarcoma. It is typically seen in patients in their teens and 20s, occurs more frequently in men than in women, and is found in the metaphyseal areas of long bones, with 50% of lesions seen about the knee joint. The distal femur is the most common site, followed by the proximal tibia, and then the proximal humerus. The only blood tests with prognostic significance are lactic dehydrogenase (LDH) and alkaline phosphatase (ALP). Patients with an elevated ALP at diagnosis are more likely to have pulmonary metastases. In patients without metastases, those with an elevated LDH are less likely to do well than are those with a normal LDH. Other laboratory tests include a complete blood cell (CBC), including a differential and platelet level, electrolytes, and liver and renal function tests.
A 14-year old girl presents due to right leg pain. She is a competitive gymnast and works out in the gym 6 days a week. Upon physical exam, there is knee pain, swelling, and tenderness at the tibial tubercle. The patient's muscles are very tight in front and behind her thigh. All laboratory findings are within normal limits. Question Highlights What is the most likely diagnosis? 1 Iliotibial band syndrome 2 Kneecap dislocation 3 Osgood-Schlatter disease 4 Patellar tendinitis 5 Sinding-Larsen-Johansson syndrome
Correct Answer: Osgood-Schlatter disease Osgood-Schlatter disease usually affects children going through a growth spurt and who are involved in jumping-centric sports. The patient presents with the appropriate symptoms of pain, swelling, and tenderness around the tibial tubercle.
An afebrile 76-year-old man has a history of pain and stiffness in his knees bilaterally and in his right hip. The pain is worse after an activity or first thing in the morning. Otherwise, he is healthy and has no other complaints. Upon physical exam, the patient has limited range of motion, joint line tenderness of knees, and mild swelling in the painful joints without warmth. 2 of 18 "tender points" are tender to palpation. A synovial fluid analysis was conducted and it was negative for urate crystals. X-rays of the knees and right hip do not show sclerosis of the bone. Additionally, a Gram stain was conducted and was negative for any bacteria. Question What is the most likely diagnosis? 1 Osteoarthritis 2 Gouty arthritis 3 Fibromyalgia 4 Avascular necrosis 5 Septic arthritis
Correct Answer: Osteoarthritis This patient presents with knee and hip pain after long use or activity, which is descriptive of osteoarthritis. Additionally, upon physical exam, the patient presents with swelling in his knees and hips, which is due to articular cartilage breaking down. This causes pain and swelling and decreased movement of the joint. In severe OA, muscle wasting can also occur.
A 6-year-old boy has gradually increasing sharp pain in his left anterior hip that seems worse at night. He is moderately overweight, but he remains active and plays baseball. There is no history of trauma. There is no redness, swelling, or fever; there is no involvement of the other hip. Ibuprofen seems to help. There is no family history of any bone or joint disease. His weight is at the 95th percentile and height is at the 50th percentile. On exam, there is mild tenderness to palpation over his left anterior hip, but the rest of the exam is normal. Laboratory studies show a normal complete blood count, hemoglobin, sedimentation rate, platelets, and C reactive protein. Plain X-rays show a small round lytic lesion surrounded by mild reactive bone formation. Question Highlights What is the most likely diagnosis? 1 Legg-Calve-Perthes disease 2 Osteomyelitis 3 Primary lymphoma of bone 4 Osteoid osteoma 5 Ewing sarcoma
Correct Answer: Osteoid osteoma
A 33-year-old Caucasian man with a history of severe type 1 diabetes presents due to a red swollen pinky toe on his left foot. He was running to answer his home phone and stepped on an exposed carpet tack 2 days ago. He washed it out and covered it as soon as he could. Since then, he has noted a significant increase in redness and severe restriction of movement of this toe; these characteristics have worsened tremendously in the last 12 hours. In addition, he admits to "feeling feverish" for the past day, and he experienced significant night sweats the previous night. He denies formally measuring his temperature. He has tried to take acetaminophen 500 mg, 2 tablets every 8 hours without any noticeable relief. Pertinent physical examination findings include an oral temperature of 102.8°F, significant erythema and edema extending the entire fifth toe of the left foot accompanied by restricted range of motion due to tenderness and swelling. There is a small amount of purulent drainage at the site of the wound. Question Highlights What is the most likely diagnosis? 1 Osteoarthritis 2 Gouty arthritis 3 Osteopenia 4 Osteomyelitis 5 Psoriatic arthritis
Correct Answer: Osteomyelitis This patient most likley has an acute case of osteomyelitis (bone infection). Possible etiologies include direct trauma, an infection from somewhere else in the body, a chronic ulcer or wound, or bacteremia. Specific patient populations are more prone to develop osteomyelitis: patients with diabetes or sickle cell disease, hemodialysis patients, IV drug users, or elderly patients. Suggestive symptoms include pain or tenderness of the affected area/bone, swelling and warmth of the area, significant fever, general discomfort, drainage of purulent material from the site, and sweating and chills.
An 89-year-old Caucasian man is brought in by his daughter due to pain in his left shin. He bumped his leg on a coffee table about 3 weeks ago; he developed some mild discomfort, bruising, and a small gash in the skin. It seemed as if he was healing well, but his condition has worsened over the past few days. Now he is moaning due to pain, and he says it hurts to walk on the leg. He describes the pain as "horrible" and an 8/10. The daughter reports the pain keeps him up at night and is unresponsive to ibuprofen and narcotic pain pills. He denies fever and chills. On physical exam, the patient is in obvious pain and is assisted to the exam table with limited weight bearing on his left leg. The lower extremities are examined; significant findings include healing and a scabbed lesion of approximately 3 cm in length across mid-tibia, with surrounding erythema and edema. Tenderness is elicited along the shin, extending well past the area of erythema. Homan's sign is negative. Distal pulses, temperature, coloration, knee range of motion, and lower extremity reflexes are symmetric and normal. Right lower extremity is normal. What is the most likely diagnosis? 1 Claudication 2 Gout 3 Osteomyelitis 4 Pes anserine bursitis 5 Stress fracture
Correct Answer: Osteomyelitis This patient presents with a likely osteomyelitis: infection of the bone tissue. In this patient's case, some bacterial spread from a relatively minor injury at the skin surface is likely responsible. Osteomyelitis can affect all age groups, but it can especially affect the elderly population and those with poor peripheral circulation. Long bones and vertebrae are most commonly affected. This patient's pain is out of proportion with exam findings. With acute osteomyelitis in adults, exam findings are often limited to local erythema and edema (with tenderness), but with exaggerated pain.
A 66-year-old woman presents with back pain. She has a 5-month history of taking prednisone and has been a smoker for the last 15 years, which she has difficulty quitting. Upon physical exam, her back is very tender upon palpation. When the patient walks, she appears very bent over and unable to stand upright. A laboratory finding on her DXA was a T score of -3.1. X-ray was conducted as well, and demonstrates a small fracture in her back. What is the most likely diagnosis? 1 Osteomalacia 2 Osteopenia 3 Osteoarthritis 4 Osteosarcoma 5 Osteoporosis
Correct Answer: Osteoporosis The clinical picture is suggestive of severe osteoporosis. The patient has risk factors of both smoking and taking prednisone. Additionally, upon physical exam, she appears to be tender upon palpation and unable to stand up. Severe osteoporosis is defined by a T score <-2.5 with a fracture, which is confirmed from both her DXA and X-ray.
A 34-year-old man presents for evaluation of 10/10 left calf pain after being run over by a car. Physical exam reveals tense calf swelling, increased pain with passive muscle stretching, and tenderness on calf palpation. Question Highlights What other findings are expected in this patient? 1 Pain out of proportion to exam 2 Normal calf compartment pressure 3 Erythematous streaks 4 A palpable calf cord 5 Groin lymphadenopathy
Correct Answer: Pain out of proportion to exam This patient presents with acute compartment syndrome after a crush injury. Compartment syndrome occurs when excessive muscle swelling increases the pressure within the fascial sheath compartment, compressing vessels and nerves. Compression of the artery leads to ischemia. Compartment syndrome is a medical emergency and can result in the loss of the limb if the pressure is not relieved immediately. Compartment syndrome is a clinical diagnosis based on the 6 Ps (pain out of proportion, pressure on palpation, paresthesia, paralysis, pallor, and pulselessness). In addition to the 6 Ps on PE, diagnosis is confirmed by obtaining the compartment pressure, in which anything above 10 mm Hg is abnormal. At 20 mm Hg, ischemia can occur, and anything above over 30 mm Hg requires an emergent fasciotomy.
A 34-year-old woman presents Monday morning with knee pain. She states that she was playing with her two children in their backyard over the weekend and fell onto solid ground, landing directly on her knees. She noted immediate and significant right knee pain. She notes significant swelling; it accompanies the knee pain, which she rates as an 8/10 on a numerical pain scale. Physical examination reveals significant obvious joint effusion and exquisite focal tenderness to palpation over the patellar area of the right knee; the left knee has no obvious abnormalities. Results of the anterior drawer, McMurray, and varus/valgus stress testing are within normal limits. Question Highlights Given the history and physical examination findings, what is the most likely diagnosis? 1 Bone cyst 2 Patellar fracture 3 Ligament tear 4 Tibial fracture 5 Femoral shaft fracture
Correct Answer: Patellar fracture The patient above has most likely experienced a right patellar fracture. Many fractures of the patella are the result of direct forces, such as striking the dashboard in a motor vehicle accident, or in the patient's scenario, falling onto a flexed knee. Patellar fractures should be high on the differential diagnosis list if a patient presents with acutely swollen knee and patella pain following trauma to that area. There will be presence of joint effusion and focal tenderness to palpation of the patella. Another feature is that the patient will be unable to extend the knee against gravity, which requires surgical repair if present. Our patient had all these signs and symptoms, minus the inability to extend the knee.
An 18-year-old man with no significant past medical history presents with pain and swelling over the upper right knee for 5 months. The pain was initially insidious, dull, and achy. Over the past several weeks, however, it has gradually become progressively more severe and unremitting, often waking the patient at night. He also notes increased swelling, warmth, and erythema. He denies a history of injuries, accidents, trauma, surgeries, or sexual encounters. His physical exam reveals a noticeable limp, reduced right knee range of motion, and localized tenderness and swelling to the distal anterior femur. Question Highlights What is the next step in the management of this patient? 1 Begin physical therapy 3 times a week for 6 weeks. 2 Begin a xanthine oxidase inhibitor and indomethacin. 3 Perform a right knee radiograph. 4 Prescribe a trial of ibuprofen and observe for 2 weeks. 5 Refer the patient for a bone scintigraphy scan.
Correct Answer: Perform a right knee radiograph. This patient's presentation is suspicious for the malignant bone tumor known as osteosarcoma. The imaging of any bone tumor should begin with a radiograph of the involved area. Such an X-ray image is often helpful in the diagnosis of bone sarcomas; osteosarcoma often has a sunburst appearance of calcification on X-ray imaging, which is virtually diagnostic. Once the diagnosis has been established, a chest X-ray, CT scan of the chest, and bone scan should be performed to evaluate for metastatic disease. As with soft tissue sarcomas, the most common site of metastasis from bone sarcoma is the lungs, warranting a chest X-ray and CT chest.
A 34-year-old man presents with a 2-day history of right ankle pain and swelling. He reports experiencing discomfort with bearing weight, ambulation, and when driving a car. On further questioning, he denies experiencing a recent trauma, although he does recall spraining his ankle approximately 1 year ago. The patient is not married and is heterosexual; he usually uses condoms as contraception, "but not every time." On physical examination, the patient's temperature is 99.9°F. His right ankle shows swelling, is warm to palpation, and reveals an effusion. With passive range of motion of the right ankle, significant pain is elicited. Question Highlights What is the most appropriate next step in the management of this patient? 1 Check serum uric acid level; if elevated, initiate therapy with indomethacin and colchicine. 2 Obtain plain radiograph of the right ankle to assess for chondrocalcinosis. 3 Obtain plain radiograph of the right ankle to assess for structural damage. 4 Perform arthrocentesis of right ankle with analysis of synovial fluid. 5 Treat with ibuprofen and have patient return to office in 1 week for follow-up.
Correct Answer: Perform arthrocentesis of right ankle with analysis of synovial fluid. The acute onset of symptoms, low-grade fever, and lack of trauma in this patient warrant a prompt evaluation. Empirical therapy will not provide a definitive diagnosis and could potentially result in a serious illness such as septic arthritis being missed. The most appropriate initial evaluation of a patient with monoarticular arthritis is arthrocentesis. Joint aspiration should be performed with aseptic technique as a part of the evaluation of every case of acute monoarthritis. Analysis of the synovial fluid includes a WBC count, differential, appropriate cultures and stains for microorganisms, and polarized-light microscopy. The WBC count in the synovial fluid is useful in distinguishing inflammatory from non-inflammatory arthritis; levels greater than 2000/mm3 are consistent with inflammation. Gonococcus (Neisseria gonorrhea) is the most common cause of infective arthritis in young adults.
A 17-year-old male high school football player presents after being tackled and slammed onto his right dominant shoulder forcefully 2 hours ago during a game. He had immediate pain but was able to continue playing. He has full active and passive range of motion, but some pain (6/10) with abduction. There is no obvious deformity, and the skin over the shoulder is intact and not tented. What initial radiographic procedure should be ordered? 1 Ultrasound 2 CT without contrast 3 Plain film radiograph 4 MRI without contrast 5 MR arthrography
Correct Answer: Plain film radiograph Assessment of acute shoulder injuries may be difficult. Standard (plain film) radiography is used to consider presence or absence of injury and to differentiate acromioclavicular from other shoulder injuries in the setting of acute injury. A minimum of two views is necessary to evaluate the injury. Views may consist of anterior-posterior, lateral, axillary, or scapular Y views.
A 75-year-old woman with a past medical history of hypertension, hyperlipidemia, and obesity presents with insidious but progressive bilateral knee pain for the past several months. She states that her pain is worsened with movement and relieved by rest. She reports "cracking" sounds in her knees upon ambulation and minimal stiffness lasting 10 minutes. Her physical exam is remarkable for bilateral knee crepitus, joint line tenderness, and an antalgic gait; there is no swelling, erythema, or warmth noted. Question Highlights What is an expected diagnostic test result in this patient? 1 Elevated erythrocyte sedimentation rate 2 Juxta-articular demineralization on x-rays 3 Negatively birefringent crystals 4 Plain radiograph subchondral sclerosis 5 Synovial fluid leukocytosis
Correct Answer: Plain radiograph subchondral sclerosis This patient is presenting with signs and symptoms consistent with osteoarthritis. Knee radiography is typically expected to reveal joint space narrowing (especially of the medial compartment), thickened, dense subchondral sclerosis, osteophytes, and bony spurring. Other findings include subchondral cyst formation and lipping of marginal bone.
A 24-year-old man with no significant PMH presents due to increasing pain in his right groin and buttock and difficulty walking. The pain has been present and worsening for about 1 month. He further reports that he had significantly injured himself (also near his right hip) in a fall while snowboarding about 3 months ago. He did not seek immediate care and had difficulty ambulating for a week, then saw a chiropractor to "put his lower back back in." The pain and gait problems eventually improved. Now he is concerned because the pain has returned with no new history of trauma. He denies any family history for musculoskeletal and rheumatologic conditions. On physical exam, the patient was noted to walk with a slight limp. Examination of the hip, buttock, and groin region reveals no edema, erythema, or ecchymosis. The exam does not produce one particular point of tenderness with palpation, passive range of motion is limited and painful, especially with forced internal rotation. A straight-leg raise against resistance elicits pain. Question Highlights What diagnostic study would confirm this patient's condition? 1 Arthroscopy 2 Bone biopsy 3 Nuclear bone scan 4 Plain radiography (X-ray) 5 Ultrasound
Correct Answer: Plain radiography (X-ray) This patient's condition is suspicious for avascular necrosis (or the more preferred term, osteonecrosis) of the hip. Avascular necrosis (AVN) is a relatively common complication, following a traumatic hip dislocation, when the vascular supply to the femoral head is damaged and bone death occurs. There are many theories about the exact mechanism; other causes such as vascular, congenital, and autoimmune diseases have been implicated with osteonecrosis. Plain radiography (X-ray) is the preferred initial test, and if the "crescent sign" is seen, AVN is confirmed. The crescent sign is a lucent (dark-appearing) crescent-shaped region of the femoral head, usually just a sliver beneath a more opaque (white-appearing) bone edge, which indicates some sclerosis and bone structure collapse upon itself.
A 73-year-old man has had shoulder pain for more than 6 months and it has kept him from sleeping on his left side and has often woken him up. He presents with an inability to actively raise his left arm that started 1 month ago, before which his shoulder pain had improved. There was no specific injury that he can recall, but he felt a pop in the shoulder a month ago while taking out the trash. Question What finding do you expect on physical exam? 1 Ecchymosis over the deltoid 2 Normal strength of all shoulder muscles 3 Negative (-) painful arc sign 4 Positive (+) drop arm test 5 Paresthesia of the median, radial, and ulnar nerves
Correct Answer: Positive (+) drop arm test This presentation indicates a rotator cuff tear; drop arm test is positive in this condition. Drop arm test is positive when the patient is unable to control shoulder adduction. History typically includes pain for several months, with difficulty sleeping on the affected side and waking from the pain. Active range of motion is limited, but passive range of motion is usually full (unless associated with a frozen shoulder prior). Painful arc sign is typically positive in rotator cuff tears. Painful arc sign is positive when the patient notes pain with active abduction beyond 90°. Nerve damage is rare with cuff tears. Weakness or reduced strength with external rotation are commonly seen on exam. No trauma occurred, and ecchymosis would be an unlikely finding.
A 54-year-old man presents with a 2-month history of left non-dominant shoulder pain. There is no history of trauma, but the pain began about a week after shoveling wet heavy snow from his 100-foot driveway. At first, the pain seemed to come and go from day to day, but it has gotten progressively worse and more constant. Pain is worse with overhead use. He cannot sleep on his left side and will wake up if he rolls over onto his left shoulder. Drop arm test is negative. Question Considering the most likely diagnosis, what other exam finding do you expect? 1 Tenderness at the acromioclavicular joint 2 Restricted active and passive motion of the shoulder 3 Positive Neer and Hawkins signs 4 Positive sulcus sign 5 Edema of the shoulder muscles
Correct Answer: Positive Neer and Hawkins signs Drop arm test is positive in rotator cuff tears. With his other symptoms, it is likely this patient has impingement syndrome. Neer and Hawkins signs are positive in impingement syndrome.
A 52-year-old woman presents with left hip pain. There is no known history of trauma to the area. Her past medical history includes Crohn's disease, type 2 diabetes, and hypertension. She is currently taking metformin, metoprolol, and mesalamine. She recently completed a prednisone taper for her Crohn's disease. She also finished a course of ciprofloxacin, which she took for a urinary tract infection. An X-ray is obtained, revealing a collapsed left femoral head. Question Highlights What medication likely contributed to her current condition? 1 Metoprolol 2 Metformin 3 Mesalamine 4 Prednisone 5 Ciprofloxacin
Correct Answer: Prednisone The patient has a collapsed left femoral head on radiography, suggesting avascular necrosis, which refers to death of bone secondary to lack of adequate blood supply. This leads to collapse of the bone (as seen on this patient's X-ray) and pain. Avascular necrosis is commonly associated with the use of corticosteroids, so prednisone is correct.
A 66-year-old man presents with monoarticular arthritis. He has stage 3 chronic kidney disease. His affected joint is warm to the touch. You suspect gout. He is allergic to aspirin. Question What anti-inflammatory should you prescribe? 1 Acetaminophen 2 Aspirin 3 Ibuprofen 4 Metformin 5 Prednisone
Correct Answer: Prednisone This patient requires an anti-inflammatory agent that will not cause a decline in renal function. Prednisone is recommended. Urate-lowering therapy such as allopurinol may also be initiated.
A 40-year-old male recreational tennis player presents with pain in his playing-side elbow since a tournament last weekend. He played five matches of singles and two matches of doubles games. He cannot even hold a pen without pain. He says he needs to get better fast because he has another tournament coming in a week. On exam, he has pain with extension against resistance of his affected side wrist. Question Highlights What treatment option should be recommended on this first visit? 1 Start strengthening exercises now to get ready for next tournament. 2 Prescribe him muscle relaxants since spasms cause the pain. 3 Tell him he should switch arms to give his normal playing arm a rest. 4 Inject him today with a long-acting anesthetic agent to relieve his pain. 5 Prescribe an NSAID of choice and recommend skipping the next tournament.
Correct Answer: Prescribe an NSAID of choice and recommend skipping the next tournament.
An 8-year-old boy sustained a puncture wound to his right foot by stepping on a nail through his sneaker 4 days ago. His mother said the wound bled profusely but the nail did not go completely through his foot. They washed the wound at home with soap and water, wrapped it in a bandage, and did not seek further care. This morning, he complained that it was very painful and his mother noted that his foot looked red and swollen. On exam, his temperature is 99°F; pulse is 114 BPM, and his BP is 104/68 mm Hg. The plantar surface of his right foot has a small 2 mm scabbed entry wound that is surrounded by a 5-6 cm area that is erythematous, swollen, and quite tender. There is a scant amount of thin seropurulent material from the entry wound on examination. Question What bacteria is most likely the cause of the suspected complication the patient is experiencing? 1 Streptococci 2 Eikenella 3 Pasteurella 4 Clostridium 5 Pseudomonas
Correct Answer: Pseudomonas Pseudomonas aeruginosa is the most common cause of foot osteomyelitis in plantar puncture wounds while wearing tennis shoes. It should be considered a likely pathogen when there is a penetrating injury through footwear, especially sneakers or from soil and manure contamination. It is believed that the penetrating object may push particles of foam contaminated with bacteria into the wound. In addition, anaerobes, Klebsiella, Bacteroides, Serratia, and Salmonella sp. are less common organisms in this setting that may also cause secondary infections.
A 12-year-old girl presents to the emergency room with worsening foot pain. 2 weeks ago, she stepped on a nail while wearing rubber-soled tennis shoes. The area was thoroughly cleaned, but she has developed worsening pain, redness, and drainage from the area. X-ray shows periosteal changes at the site of the wound. A wound culture is obtained. Question Highlights What organism is most likely to grow on culture for this specific mechanism of injury? 1 Staphylococcus aureus 2 Escherichia coli 3 Pseudomonas aeruginosa 4 Serratia marcescens 5 Salmonella enterica
Correct Answer: Pseudomonas aeruginosa Pseudomonas aeruginosa is the most common cause of osteomyelitis when a penetrating object passes through the sole of the shoe, though Staphylococcus aureus is the most common cause of osteomyelitis overall.
Parents bring their 4-year-old daughter in because of knock-knee. She is otherwise healthy, and her height is in the 50th percentile for age. On examination, she has about 10 degrees of valgus. Question Highlights What should you recommend? 1 X-rays of the lower extremities 2 Reassurance that this is normal for age 3 Orthopedic shoes 4 Long leg braces 5 Lab tests for markers of bone turnover
Correct Answer: Reassurance that this is normal for age Children are normally bowlegged (genu varum) at birth. By 12-18 months, the legs become straight. Then children develop knock-knee (genu valgum), which is maximal by age 3-4. This averages 10-15° at this age. The legs then spontaneously "straighten" to 5-10° of valgum, which is the average for adults. This child is within the normal range, and the parents should be reassured.
A 33-year-old man with no significant past medical history presents with a 2-month history of persistent right knee pain. The knee pain is located in the anterior part of the knee; "behind the kneecap," according to the patient. Pain is worse as he descends stairs, performs squatting maneuvers, and sits for excessive periods of time. He is an avid runner and states that running also increases pain. He denies any trauma, falls, accidents, or prior surgeries. He further denies any fever, chills, insect bites, rashes, effusions, grinding, popping, or clicking sensations in the knee. He denies any hip or ankle pain. The physical exam reveals tenderness to palpation along the medial undersurface of the right patella and a positive patellar apprehension test. The anterior and posterior draw tests, McMurray's test, and Apley's compression and distraction tests are all negative. Question Highlights What is the most appropriate clinical intervention for this patient at this time? 1 Weight gain 2 Reduction of running exercises 3 Prescription for narcotic analgesia 4 Referral for knee arthroscopy 5 Cast immobilization
Correct Answer: Reduction of running exercises This patient's most likely diagnosis is patellofemoral pain syndrome. Clinical interventions include relative rest, a reduction of running pace and mileage, and avoidance of squats, lunges, and running uphill and downhill. Other interventions include quadriceps strengthening and NSAID medications. Surgical interventions are indicated if symptoms persist beyond 6 months, if a rehabilitation program fails, and if other causes of knee pain have been excluded. Weight loss is recommended to decrease the stress on the patellofemoral joint. Knee orthotics may be beneficial.
A 73-year-old man presents with the inability to actively raise his left non-dominant arm to retrieve plates from the kitchen cabinet. He further describes the inability to retrieve any objects with his left hand/arm because of the limited range of motion. He is worried because he is the only driver in his household, and he does not want to lose his driving privileges. He has a longstanding history of chronic shoulder impingement syndrome. On exam, he is found to have a (+) positive drop arm test. Question Highlights What initial treatment is recommended for the most likely diagnosis? 1 Immediate subacromial steroid injection 2 Immediate surgical repair 3 Order CT of the left shoulder 4 Refer for physical therapy 5 Place in shoulder immobilizer
Correct Answer: Refer for physical therapy Rotator cuff tears (RCT) are a common clinical issue in geriatric patients, and there has been a long debate on how to approach restoring shoulder function in these patients. Treatment options usually consist of non-surgical and surgical options. Many times the majority of patients are initially placed on a trial of conservative therapy, which would include physical therapy. Refer this elderly patient with a rotator cuff tear to physical therapy to attempt regaining range of motion by stretching and strengthening the remaining rotator cuff.
A 21-year-old male presents with a 3-week history of pain in his left shoulder. The pain began when he started spring training and has gradually worsened. In addition, the patient is now having difficulty lifting the arm above his head. He denies injury to the shoulder itself; he states that ibuprofen provides some relief. On physical exam, the patient has tenderness to palpation of the lateral left shoulder just under the acromion, limited abduction of the left shoulder, and a negative drop-arm sign and crossover test. Question Highlights What is the next appropriate step in the management of this patient? 1 Refer for physical therapy 2 Manage pain with opiate analgesics 3 Prescribe a muscle relaxant 4 Obtain MRI of the shoulder 5 Refer for orthopedic surgical therapy
Correct Answer: Refer for physical therapy This patient most likely has rotator cuff tendinitis. Repetitive movements of the shoulder, such as throwing or swimming, can lead to inflammation of the tendons of the rotator cuff muscles. A patient with a rotator cuff tendinitis will have tenderness with palpation of the area just under the acromion on the affected side and an inability to lift the arm above shoulder level (abduction and external rotation). For most rotator cuff disorders, rest from activities that increase pain is helpful. In addition, NSAIDs and moist heat are first-line treatment for these conditions. Corticosteroid injections and physical therapy can also be used in patients who do not get full relief from NSAIDs, or in those who have recurring issues. This patient has tried NSAIDs and has continued to have pain and inability to lift the arm above the shoulder, so physical therapy is indicated.
A 27-year-old man is admitted to the hospital following a motor vehicle accident. He sustained lacerations to his arms bilaterally and has fractures of the right tibia and fibula. A cast is placed and the patient is scheduled for surgery the following day. A few hours after the cast is placed, he develops severe pain; the pain is unresponsive to several doses of intravenous morphine. His pain increases when he extends his right leg. Peripheral pulses are weak but present. Question Highlights Based on the history and physical, what should be done next? 1 Take X-ray of the tibia and fibula. 2 Perform MRI of the right leg. 3 Remove cast and check compartment pressure. 4 Perform ultrasound of the lower extremity. 5 Monitor and continue giving narcotics.
Correct Answer: Remove cast and check compartment pressure. The patient should have the cast removed and compartment pressure measured. This patient is showing signs of compartment syndrome. Compartment syndrome develops when there is an accumulation of pressure within a muscle compartment. It typically occurs following trauma to a limb. Patients typically present with pain unrelieved by analgesia as well as pain with extension of the involved limb. As the swelling within the compartment worsens, patients develop absence of pulse, paresthesias, pallor, and poikilothermia. The compartment pressure should be checked; if it is elevated, the patient should be taken for fasciotomy.
A 28-year-old woman presents with severe right ankle pain. Further questioning reveals the patient is unable to move her ankle. She states that it is exquisitely tender and the pain worsens with movement or when attempting to bear weight. She states that she was immediately unable to take any steps because of how substantially painful the right leg is. During the physical examination, the patient continues to be unable to bear weight on the injured leg. You note that the right ankle is painful to even light palpation throughout the malleolar zone. The skin is a healthy color and there are no signs of lacerations or tenting. Pulses are palpable. The entire area of the right ankle is swollen and appears dislocated. The left leg, foot, and ankle are unremarkable. Question Highlights Given the history and physical examination, what is this patient most likely suffering from? 1 Right inversion ankle sprain 2 Right eversion ankle sprain 3 Right ankle strain 4 Right ankle fracture 5 Right ankle hematoma
Correct Answer: Right ankle fracture The correct answer for this patient scenario is a right ankle fracture. Since there is significant swelling bilaterally, both the tibia and the fibula are very highly likely to be fractured. Key features of an ankle fracture include pain, swelling/signs of bruising after the traumatic event, apparent deformity, and the inability to bear weight; this patient fits all of these components. Ottowa Ankle Rules have been found to determine with high sensitivity if an ankle fracture can be ruled out. The rules state an ankle X-ray is only required if: There is any pain in the malleolar zone; and, Any one of the following: Bone tenderness along the distal 6 cm of the posterior edge of the tibia or tip of the medial malleolus, OR Bone tenderness along the distal 6 cm of the posterior edge of the fibula or tip of the lateral malleolus, OR An inability to bear weight both immediately and in the emergency department for four steps.
A patient presents with a motor deficit on the right side of the body, decreased sensation and pain on the left side of the body, and diminished vibratory and position sense on the right side of the body. What type of spinal cord lesion is present? 1 Left hemisection of spinal cord 2 Bilateral posterior column lesion 3 Anterior spinal artery lesion 4 Syringomyelia in cervical spinal cord 5 Right hemisection of spinal cord
Correct Answer: Right hemisection of spinal cord A hemisection of the spinal cord results in a Brown-Sequard's syndrome. The characteristic deficits of the Brown-Sequard's syndrome include a motor deficit and deficit of position sense and vibratory sense ipsilateral to the side of the lesion. There is a pain and temperature deficit that is contralateral to the side of the lesion. These deficits occur because the fibers in the cortical spinal tract and in the posterior columns decussate above the level of the spinal cord. The fibers that serve pain and temperature sensation decussate upon entering the spinal cord.
A 73-year-old man has had a history of pain in that shoulder for over 6 months that kept him from sleeping on the left side; the pain would wake him often. He presents with an inability to actively raise his left non-dominant arm, which began a month ago after his shoulder pain improved. There was no specific injury he can recall, but he felt a pop a month ago while taking out the trash. On inspection, you notice the back of the shoulder appears sunken compared to the other shoulder. Question Highlights What is the most likely diagnosis? 1 Rotator cuff tear 2 Impingement syndrome 3 Adhesive capsulitis 4 Glenohumeral dislocation 5 Proximal humerus fracture
Correct Answer: Rotator cuff tear Elderly patients with rotator cuff tears typically report a prodrome of chronic shoulder pain for several months and an episode where they feel a pop when performing some heavier activity than usual. A sunken shoulder indicates atrophy due to the tear.
A 21-year-old man presents with a 3-month history of pain in his left shoulder. He is a left-handed pitcher for his college baseball team. The pain began when he started spring training a few months ago, and it has gradually worsened since that time. In addition to the pain, the patient is now having difficulty lifting his left arm above his head. He denies any injury to the shoulder itself, and he states that ibuprofen provides some relief. On physical exam, the patient has tenderness to palpation of the lateral left shoulder just under the acromion, limited abduction of the left shoulder, a negative drop-arm sign, and a negative crossover test. Question Highlights What condition best explains the patient's symptoms? 1 Bicipital tendinitis 2 Acromioclavicular arthritis 3 Rotator cuff tendinitis 4 Rotator cuff tear 5 Adhesive capsulitis
Correct Answer: Rotator cuff tendinitis The patient above is most likely suffering from rotator cuff tendinitis. Distinguishing between the various soft tissue disorders that can cause shoulder pain—tendinitis, subacromial bursitis, and tears—can be difficult because these conditions can cause similar symptoms and can often co-exist. Knowledge of musculoskeletal anatomy and physiology can help in making a more precise diagnosis. The muscles of the rotator cuff include the supraspinatus, infraspinatus, teres minor, and subscapularis, otherwise known as the SITS muscles. The tendons of three of the rotator cuff muscles form a musculotendinous unit where they insert into the proximal humerus. Repetitive movements of the shoulder, such as throwing or swimming, can lead to inflammation of the tendons of the rotator cuff muscles. The most commonly affected muscle is the supraspinatus, which inserts directly under the acromion. A patient with a rotator cuff tendinitis, in addition to the inability to lift the arm above shoulder level (abduction and external rotation), will therefore have tenderness with palpation of the area just under the acromion on the affected side.
A 35-year-old man presents with right shoulder pain that is becoming progressively worse. Although he visits the gym 3 times a week, over the past month he has not been able to increase the amount of weight he lifts secondary to the shoulder pain. He has not tried anything to alleviate the pain. He reports that the pain is at its worst at night while he is trying to sleep. He also reports pain while in the shower washing his hair, or when using the shoulder press machine at the gym. He denies any history of recent trauma or sports-related injury. Upon questioning, however, he reports that about 1 month ago he and his wife painted their entire house in one weekend. On exam, there is no notable swelling, atrophy, redness, or bruising of the shoulders. Point tenderness is noted over the right lateral deltoid muscle. Active ROM of the right shoulder at 80° of abduction elicits pain. Patient has a negative drop arm test, negative apprehensive test, and a positive Neer impingement test of the right shoulder. Question Highlights What is the most likely diagnosis? 1 Acromioclavicular sprain 2 Adhesive capsulitis 3 Fracture of the clavicle 4 Glenohumeral dislocation 5 Rotator cuff tendonitis
Correct Answer: Rotator cuff tendonitis
A 15-year-old boy presents for a routine physical. He reveals a 1-month history of mildly painful swelling of the anterior superior left shin, unaccompanied by fever, erythema, or joint problems. He plays football for his high school and has been assisting his father in in a project that entails considerable kneeling that worsened the pain in the affected area. Examination is unremarkable except for mild slightly tender swelling of the left anterior shin approximately 5 cm below the knee. Radiographs of the left knee showed mild irregularity of the tibial tubercle. 1 Aspiration of the left knee joint 2 Bone biopsy 3 Routine symptomatic treatment 4 Rheumatoid factor 5 Serial knee X-rays every 4 weeks
Correct Answer: Routine symptomatic treatment The diagnosis is Osgood-Schlatter disease, an inflammation of the tibial tuberosity. Once the significant conditions are eliminated, treatment is supportive and symptomatic: rest, ice, wrapping, elevation, NSAIDs, and physical therapy.
A 55-year-old woman presents with weakness, bone pain, and lethargy. A 24-hour urine sample reveals a spike of M protein. Based on the most likely diagnosis, what additional finding on bone marrow aspirate would support the diagnosis? 1 Teardrop-shaped erythrocytes 2 Lewy bodies 3 Schistocytes 4 "Starry sky" pattern 5 Russell bodies
Correct Answer: Russell bodies Russell bodies are the accumulation of immunoglobulin in plasma cells: a characteristic finding in patients with multiple myeloma.
A 3-year-old African American girl with sickle cell disease presents with a limp; her mother first noticed the limp yesterday morning. The girl has not had any fever, nausea, or vomiting, but she is in pain. The girl's sickle cell disease has been relatively mild; she is fully immunized and takes prophylactic oral penicillin 125 mg 2 times a day. On exam, she is afebrile. There is pain in the left hip on internal and external rotation, but there is no definite point tenderness. Exam of the left knee and right hip are negative. You are concerned about septic arthritis, and you plan further workup. Question Highlights What organism are you most concerned about? 1 Staphylococcus aureus 2 Streptococcus species 3 Haemophilus influenzae type b 4 Salmonella species 5 Neisseria gonorrhoeae
Correct Answer: Salmonella species The correct response is Salmonella species. Choosing the initial antibiotic therapy in suspected osteomyelitis depends on the likely bacterial pathogens based on the age of the patient, the results of Gram staining of the joint aspirate, and consideration of any special circumstances, such as this patient's sickle cell disease. In otherwise healthy children under 5, the most likely causative organism in osteomyelitis is S. aureus. Other common causes in patients without sickle cell disease are Streptococci and H. influenzae type b in children that have not been immunized.
A 33-year-old woman presents with joint pain and morning stiffness that lasts more than an hour. Her past medical history is significant for postpartum depression 2 years ago. On examination, you find her BMI is 18 and she displays signs of symmetrical small joint inflammation. You are waiting for X-rays and labs to confirm the diagnosis. In the meantime, she is asking about risk factors, particularly the possibility of worsening her joint condition if she becomes pregnant again. Question Highlights Based on the most likely diagnosis, what is the primary risk factor in this patient's case? 1 Age 2 Sex 3 Depression 4 BMI 5 Pregnancy
Correct Answer: Sex The correct response is sex. Rheumatoid arthritis affects women 3 times as often as men.
A 56-year-old man wakes up in the morning to find that he has a swollen, red, and painful big toe on his left foot. He had been on a cruise to the Bahamas 2 days earlier, and he spent much of the time eating and drinking. He normally has 1 glass of wine with dinner on the weekends, but his alcohol consumption increased substantially while on the cruise. He also did a great deal of walking in an attempt to make up for his excesses. He goes into his physician's office, and tests are run. An X-ray shows no acute fracture, and his vital signs are within normal limits. Blood work shows an increase in uric acid, but it is otherwise normal. He begins treatment and feels better within 24 hours. Question Highlights What most accurately describes the typical presentation of this disease? 1 Polyarticular arthritis 2 Small lower extremity joints 3 Caucasians more likely than African Americans 4 Women more likely than men 5 Under age 25
Correct Answer: Small lower extremity joints Gout presents as acute monoarticular arthritis in 90% of patients. In early gout, usually only 1 or 2 joints are involved. Usually, they are the smaller lower extremity joints. Podagra, or inflammation of the first metatarsophalangeal joint, is the initial joint manifestation involved in about half of all cases, but podagra may also be observed in patients with pseudogout, reactive arthritis, gonococcal arthritis, psoriatic arthritis, and sarcoidosis. The attacks usually begin abruptly and can reach maximum intensity in 6-12 hours. The joints are red, hot, and extremely tender. Untreated, the characteristics of gout change over time. The attacks become more polyarticular. Although more joints may become involved, inflammation in a given joint may become less intense. Attacks occur more frequently and last longer. Eventually, patients may develop a chronic polyarticular arthritis, which can be symmetrical and resemble rheumatoid arthritis. Tophi, which are collections of uric acid crystals in the soft tissues, occur frequently in untreated patients. They can be found in multiple locations, including the fingers, toes, in the olecranon bursae, and along the olecranon (where they may appear to be rheumatoid nodules). Tophi tend to develop after 10 years in untreated patients who develop chronic gouty arthritis.
A 63-year-old Caucasian woman presents to review her recent imaging results. She has a past medical history of gastroesophageal reflux disease and mild seasonal allergies. She takes a daily omeprazole and an as-needed cetirizine. She smokes 15 cigarettes daily and has done so for the past 33 years. Family history is significant for her mother having a broken hip when she was 70 and an aunt who had 2 broken vertebrae at ages 68 and 71. The dual-energy X-ray absorptiometry (DXA) results are as follows: T-score lumbar spine: -2.4, T-score total hip: -2.5, T-score femoral neck: -2.6. Question Highlights Considering the patient's diagnosis, what would be the most crucial lifestyle modification she could make to improve her future results? 1 Decreasing caffeine intake 2 Increasing oral intake of calcium 3 Increasing oral intake of vitamin D 4 Smoking cessation 5 Strength training physical activity
Correct Answer: Smoking cessation Smoking cessation is the correct response. In the pathology known as osteoporosis, the ultimate issue that can occur is bones becoming weak or brittle to the point where a fall or even minor stresses such as coughing could cause fractures. This chronic disease state is especially in common in Caucasian and Asian women past menopause. Certain specific risk factors increase a patient's risk of developing osteoporosis and therefore influence how successful treatment is if/when it is initiated. These risk factors include both smoking and alcohol consumption. Other common causes include aging, high-dose corticosteroid administration, and sex hormone deficiency. Osteoporosis is often asymptomatic until a fracture has occurred. There are many non-invasive techniques that can be used to assess if osteoporosis is present and how severe the disease state currently is in the patient. These studies include dual-energy X-ray absorptiometry (DXA), single-energy X-ray absorptiometry, (SCA), quantitative CT, and ultrasound. The DXA is usually extremely accurate in diagnosing osteoporosis and has become the standard for evaluating bone density; it is what was utilized in our patient above. Patients with a T score that is -2.5 or more in the spine, total hip, or femoral neck are diagnosed as being osteoporotic. Our patient's T scores fell just around that cut-off number. Although not completely understood, the use of cigarettes over a long period of time has been found to have a detrimental effect on bone mass. It is felt that perhaps these consequences are due to toxic effects on the osteoblasts or possibly more indirectly via estrogen metabolism. Active tobacco use contributes to the progression of osteoporosis and therefore should be considered the very first modification this patient should make in regard to her diagnosis. Smoking can also have secondary effects on bone mass, including respiratory illness, frailty, decreased exercise, poor nutrition, and the need for additional medications that may affect bone mass in the long run (e.g., glucocorticoids).
A 68-year-old man presents due to lower back pain. The patient reports the pain has been present for the past 3 months and seems to be getting worse. Upon physical exam, the patient appears to get relief of pain when bending forward. Upon standing and extension of the lumbar spine, the patient reports pain. He denies decreased range of motion in the shoulders, neck, and hips. The spine is not in an S or C shape. T2 weighted imaging shows disc degeneration. X-rays show symmetrical joint spaces. C-reactive protein (<1.0 mg/L) and ESR (<40 mm/hr) are in normal range. Gram stain is negative for Staphylococcus aureus. Question Highlights What is the most likely diagnosis? 1 Spinal stenosis 2 Osteoarthritis 3 Polymyalgia rheumatica 4 Scoliosis 5 Septic arthritis
Correct Answer: Spinal stenosis
A 22-year-old woman presents for evaluation of knee pain. The patient appears in moderate distress and is unable to fully bear weight as she moves to the exam table. Her physical exam reveals a swollen and tender left knee. She is tender at the lateral femoral condyle, lateral tibial plateau, and tibiofemoral joint line. Lachman and anterior drawer tests are positive. Posterior drawer, Apley grind, patellar grind, and McMurray tests are negative. There are no sag or apprehension signs. Valgus and varus stress tests are normal. An X-ray shows no fractures. Question What inciting event is most likely in this patient's history? 1 Dashboard knee injury in a car collision 2 Fever and prior Neisseria gonorrhoeae infection 3 Frequent squatting and climbing stairs 4 Sudden landing playing basketball 5 Lateral blow to knee
Correct Answer: Sudden landing playing basketball This patient's physical exam findings demonstrate a deficiency in the anterior knee, specifically the anterior cruciate ligament (ACL). ACL injuries most often occur acutely while playing sports, but they can occur with other mechanisms. Female sex is a risk factor. The patient may give a history of hearing a "pop," acute pain, and buckling or instability of the knee. Sudden landing after jumping during basketball is the most likely to produce an ACL injury.
A 23-year-old woman presents with a rash and swollen joints. She had been healthy previously, and the only medication she takes is acetaminophen. A review of systems includes recent fevers and a 5 lb weight loss in the past month; she is also experiencing photosensitivity and hematuria. She denies oral ulcers, nasal congestion/discharge, ear pain, pleuritic symptoms, chest pain, neural symptoms, bruising, and bleeding. On physical exam, her temperature is 101°F; her blood pressure is 130/85 mm Hg. She has a malar rash as well as diffusely swollen, warm, and tender joints. Her cardiopulmonary exam is normal. She has no costovertebral angle tenderness. Trace bipedal edema is noted. Diagnostic studies reveal: Erythrocyte sedimentation rate 50 mm/hour Hemoglobin 11 g/dL Anti-double-stranded DNA antibodies Positive Anti-nuclear antibody Positive Anti-histone antibody Positive Platelet count 200,000 cells/mL White blood cell count 12,000 cells/mm3 Chest X-ray Normal Urinalysis No bacterial growth. Numerous dysmorphic red blood cells. 10 white blood cells/high power field. Red blood cell casts. 510 mg albumin/g creatinine. Complement 3, 4 Both reduced Anti-neutrophil cytoplasmic antibody Negative Question Highlights What is the most likely diagnosis? 1 Drug-induced lupus 2 Systemic lupus erythematosus 3 Granulomatosis with polyangiitis 4 Microscopic polyangiitis 5 Henoch-Schönlein purpura
Correct Answer: Systemic lupus erythematosus This woman most likely has systemic lupus erythematosus (SLE), given her symptoms of hematuria, photosensitivity, and arthritis, in addition to the findings of urinary red cell casts, proteinuria, and hypocomplementemia. The most common clinical features included arthritis, hematological abnormalities, malar rash, photosensitivity, renal involvement, oral ulcers, serositis and neurological involvement. Renal involvement is frequent in SLE, and it may include a variety of glomerular diseases. Hypocomplementemia may occur due to complement consumption in the glomeruli. Focal proliferative and diffusely proliferative glomerulonephritis are two forms of glomerulonephritis that might be seen in this patient if biopsied. If proven, she might be treated with immunosuppressive therapy, such as prednisone and cyclophosphamide. Patients with SLE nephritis may recover, relapse, progress to a different form of lupus nephritis, or progress to end-stage renal disease.
A 17-year-old male high school football player presents after being tackled and slammed onto his right dominant shoulder forcefully 2 hours ago during a game. He had immediate pain but was able to continue playing. He has full active and passive range of motion, but some pain (4/10) with abduction. There is no obvious deformity, and the skin over the shoulder is intact and not tented. Question Highlights What finding do you expect on physical exam? 1 Tenderness over the greater tuberosity 2 Tenderness over the acromioclavicular joint 3 Crepitus with motion 4 Decreased radial pulse 5 Paresthesia over the elbow
Correct Answer: Tenderness over the acromioclavicular joint Acromioclavicular (AC) separations typically occur as a result of a direct blow to the very lateral shoulder—usually a fall onto the shoulder. This AC separation is probably a Type I. Type I will have tenderness over the AC joint, no deformity, and fair motion with some pain, but the patient may still be able to play football with the shoulders padded and limited contact. Pain mainly occurs with abduction. They will typically hold their arm adducted and supported.
A 21-year-old man presents with a 3-month history of pain in his left shoulder. He is a left-handed pitcher for his college baseball team and states that the pain began at spring training and has gradually worsened since that time. In addition to the pain, the patient is now having difficulty lifting his left arm above his head. He denies any injury to the shoulder itself and states that ibuprofen provides some relief. You suspect rotator-cuff tendinitis. Question Highlights What physical exam finding would you expect to find in this patient? 1 A positive crossover test 2 Tenderness to palpation along the anterior aspect of the left humerus 3 Tenderness to palpation just under the acromion 4 A positive Speed's test 5 Palpable crepitus during range-of-motion maneuvers
Correct Answer: Tenderness to palpation just under the acromion The correct response is tenderness to palpation just under the acromion. The muscles of the rotator cuff include the supraspinatus, infraspinatus, teres minor, and subscapularis, otherwise known as the SITS muscles. Three of these muscles—the supraspinatus, infraspinatus, and teres minor—insert on the greater tuberosity of the humerus. Repetitive movements of the shoulder, as with throwing or swimming, can lead to inflammation of the tendons of the rotator cuff muscles; the most commonly affected muscle is the supraspinatus, which inserts directly under the acromion. Classically, a patient with rotator cuff tendinitis will have tenderness to palpation just under the acromion on the affected side as well inability to lift the arm above shoulder level (abduction and external rotation).
A 32-year-old man presents with pain in his right shoulder. He tells you he plays softball every weekend and does a lot of shoveling around his house. The shoulder has been sore for some time, but now it hurts to the point where he tries to avoid using it. The drop arm test is negative, and there is no redness, warmth, or obvious swelling, but the patient is unable to lift his arm up to 90° without pain. Question Highlights What is the most likely diagnosis? 1 Bursitis 2 Fibromyalgia 3 Gout 4 Rotator cuff tear 5 Tendinitis
Correct Answer: Tendinitis Tendinitis is an inflammation of the tendon tissue, or the tendon sheath (tenosynovitis). The cause is often unknown. As the vascularity of tendons decreases with age, the incident of tendinitis will increase. Symptoms usually include painful tendons on movement. Swelling can be seen or felt on palpation. Along the tendon, localized tenderness of variable severity is present on palpation. Generally, there is not the warmth or redness of bursitis.
A 32-year-old woman presents with a 3-month history of right hand pain and paresthesia of the thumb, index, and middle fingers. She works in a retail sales office and spends much of the day typing. Question Highlights What associated physical finding may indicate an advanced case of this suspected diagnosis? 1 Thenar atrophy 2 Hypothenar atrophy 3 Weakness on extension of the DIP joints 4 Decreased gap to 2 mm on 2-point discrimination test 5 Weak radial artery pulse
Correct Answer: Thenar atrophy When compression of the median nerve is severe and long-standing, atrophy of the thenar muscles—which are supplied by the motor branch of the median nerve—can occur, indicating possible denervation of the muscles.
A 24-year-old man presents with radial-sided wrist pain following a fall on his outstretched hand 3 days ago. He did not seek immediate medical attention due to a lack of swelling. Despite self-treatment with ice and analgesics, his wrist still hurts, especially with ulnar deviation and while trying to open a jar or grip the steering wheel of his car. On examination, his tenderness seems to be in the anatomic snuffbox of the wrist. Radiographs are normal in all views. Question What is the proper treatment for this patient at this time? 1 Elastic bandage for support, continue ice and analgesics, follow up PRN 2 Wrist volar splint, continue ice and analgesics, follow up in a month 3 Thumb spica splint and referral to an orthopedic surgeon 4 Immediate referral to an orthopedic surgeon 5 Routine referral to an orthopedic surgeon
Correct Answer: Thumb spica splint and referral to an orthopedic surgeon If a patient with a suspected scaphoid fracture has radiographs that are read as "normal," but clinically there is a fracture, place the patient in a long arm thumb spica splint and refer the patient to an orthopedic surgeon.
You are evaluating a 16-year old Caucasian male high school junior (his baseball team's starting shortstop) for left hamstring soreness 4 days in duration. He felt a slight pull during infield practice. He was late that day and skipped his normal warmup and stretching routine. Despite rest, the soreness has persisted. He would like some medication for the discomfort but noted that several oral analgesics and anti-inflammatories had caused gastric upset in the past. Past medical history is otherwise unremarkable, growth and development are normal, and immunizations are current. Vital signs are normal. Examination reveals mild tenderness to palpation of the left hamstrings, aggravated by extension at the knee. Question Highlights What is the best choice of drug therapy? 1 Intramuscular methylprednisolone 2 Oral hydrocodone/acetaminophen 3 Oral prednisone 4 Topical diclofenac gel 5 Oral ibuprofen
Correct Answer: Topical diclofenac gel Pharmacological management of minor sports injuries such as strains and sprains with topical NSAIDs provides a viable alternative to oral medication. Topical NSAIDs are generally effective in relieving the pain of minor sprains and strains when applied to the affected area and are well-tolerated, even by those who have experienced unacceptable side effects to oral NSAIDs, such as gastrointestinal upset. Topical NSAIDs have anti-inflammatory, antipyretic, and analgesic qualities. Side effects are generally limited to local site reactions, such as erythema and pruritus.
A 49-year-old Caucasian man presents with pain in his left lower extremity. During questioning, the patient states that while doing construction work 3 months ago, he jumped from an elevated height of 6 feet. Ever since this episode, he has noted increased issues with left-sided hip and knee pain. He describes the pain as radiating into the left groin and front middle thigh area. The pain is relieved with sitting and aggravated by walking and climbing up stairs. The patient denies any paresthesias, numbness, bowel or bladder dysfunction, fever, night sweats, or chills. Pertinent medical history includes a 20-year extensive history of alcohol. A radiograph interpretation shows the presence of a crescent sign and marked irregularity of the left femoral head with sclerosis. Question Highlights Considering the most likely diagnosis, what would be the ultimate clinical intervention necessary for this patient? 1 Total hip replacement 2 Vascularized bone grafting 3 Daily oral bisphosphonates 4 Daily oral corticosteroids 5 Non-vascularized bone grafting
Correct Answer: Total hip replacement Avascular necrosis, sometimes also referred to as osteonecrosis, is osseous cell death resulting from vascular compromise. Commonly affected sites include the proximal or distal femoral head, ankle, shoulder, and elbow. Causes include corticosteroid use, alcoholism, trauma, systemic lupus erythematosus, pancreatitis, gout, and sickle cell disease. Radiographic findings include mild density changes early on and sclerosis. A more progressed disease state leads to the pathognomonic crescent sign (subchondral collapse). Later disease shows joint-space narrowing and degenerative changes in the acetabulum. The age of this patient would make the treating healthcare provider pursue conservative efforts to help preserve the affected joint. The progressive process of this disease requires total hip replacement. The initial treatment plan usually includes avoidance of weight bearing of the affected joint for several weeks.
A 34-year-old woman presents to your clinic with a 3-month history of a mildly tender mass on the dorsocentral aspect of her dominant right wrist. She says the lump "comes and goes," but this time it has stayed and become tender. She denies any known trauma of the wrist or hand. Question What examination technique would help to show that this mass is a ganglion cyst? 1 Checking the active range of motion of the wrist 2 Palpating the mass for size and shape 3 Obtaining radiographs of the wrist 4 Transilluminating the mass 5 Palpating the mass for tenderness
Correct Answer: Transilluminating the mass A prominent ganglion cyst of the dorsal wrist will transilluminate, while a solid tumor will not. The active range of motion is usually not affected with a ganglion cyst except for tenderness that may limit the motion. Although ganglions are usually smooth on the surface, tumors can be too. Radiographs are taken to rule out any bony pathology, but they won't reveal the ganglion. MRI is used for locating the mass and its origin. Most ganglions and tumors are tender to firm palpation or pressure.
A 12-year-old boy presents with a 3-month history of intermittent moderate right knee pain and mild right hip pain increasing in frequency and severity over the past week. There is no history of trauma or associated symptoms. He has begun to experience stiffness in the right hip and has developed a limp. Examination reveals normal vital signs, height 64", weight 182 lb (>95th percentile). He walks with an antalgic gait with the right foot externally rotated. There is pain on passive range of motion of the right hip, but not the right knee. Internal rotation of the right hip is limited. External rotation of the right hip with the knee flexed produces external rotation of the right lower leg. Radiographs of the right knee are normal, but radiographs of the right hip reveal a moderate increase of the angle of the femoral head to shaft. Question Highlights What is the best next step to treat the suspected diagnosis in this patient? 1 Rest, ice, compression, and elevation 2 Referral to physical therapy 3 Ibuprofen 400 mg every 4 hours 4 Application of knee brace 5 Urgent referral to orthopedic surgery
Correct Answer: Urgent referral to orthopedic surgery Slipped capital femoral epiphysis (SCFE) is characterized by displacement of the femoral head through the physeal plate. This usually appears in overweight male teenagers and presents with hip or knee pain, hip stiffness, altered gait, and decreased range of motion. Management involves avoidance of weight-bearing activity and urgent referral to an orthopedic surgeon. Surgery should be performed imminently to relieve compromise of the vasculature of the capital femoral epiphysis. Unless surgically corrected, there is an increased risk of avascular necrosis of the femoral head.
A 24-year-old man presents due to increasing pain in his right groin and buttock and difficulty walking. The pain has been present and worsening for about 1 month. He further reports that he had quite significantly injured himself also near his right hip in a fall while snowboarding about 3 months ago. He did not seek care immediately and had difficulty ambulating for a week, then saw a chiropractor to "put his lower back back in." The pain and gait problems eventually improved. Now he is concerned because the pain has returned with no new history of trauma. He is otherwise healthy and takes no regular medicines. He denies any chronic health conditions and has an unremarkable family history for musculoskeletal and rheumatologic conditions. On physical exam, the patient was noted to walk into the exam room with a slight limp. Examination of the hip, buttock, and groin region reveals no edema, erythema, or ecchymosis. The exam does not produce one particular point of tenderness with palpation, but pain is elicited with right hip motions, particularly internal rotation. Diagnostic testing revealed a "crescent sign" and confirmed the suspected condition. The patient will be scheduled for definitive treatment within a week. Question What approach is best to prevent further complications and maintain his current health until definitive treatment occurs? 1 Begin aggressive physical therapy. 2 Initiate antibiotics. 3 Initiate bisphosphonate therapy. 4 Initiate high-dose corticosteroids. 5 Utilize crutches for any ambulation.
Correct Answer: Utilize crutches for any ambulation. This patient's condition is suspicious for avascular necrosis (or the more preferred term, osteonecrosis) of the hip. Avascular necrosis (AVN) is a relatively common complication following a traumatic hip dislocation when the vascular supply to the femoral head is damaged and bone death occurs. There are many theories about the exact mechanism; other causes, such as vascular, congenital, and autoimmune diseases, have been implicated with osteonecrosis. If the "crescent sign" is seen on plain radiography (X-ray), AVN is confirmed. Definitive treatment is surgical, with a variety of approaches, from core decompression to bone grafting. Until surgery occurs, symptomatic patients should utilize crutches for any ambulation to prevent collapse of the femoral head.
A 16-year-old boy is currently an inpatient 7 days after a motor vehicle accident that resulted in multiple lower extremity fractures. The fractures were surgically corrected recently. He is currently immobilized below the waist, but casts will not be placed until the majority of his edema has resolved. He is stable, the edema is resolving, and he is recovering well, with the exception of new onset of increased edema and erythema over his left thigh. X-rays show evidence of acute osteomyelitis. The patient does not have any allergies. A culture from the infected bone has revealed methicillin-resistant Staphylococcus aureus. Question Highlights What treatment regimen is the best choice to treat his infection following surgical debridement? 1 Nafcillin (Unipen) 2 g IV q 4-6 hours for 6 weeks 2 Vancomycin (Vancocin) 1 g IV q 12 hours for 6 weeks 3 Oxacillin (Bactocill) 2 g IV q 4-6 hours for 2 weeks 4 Ciprofloxacin (Cipro) 400 mg IV q 12 hours for 2 weeks 5 Linezolid (Zyvox) 600 mg IV q 12 hours for 2 weeks
Correct Answer: Vancomycin (Vancocin) 1 g IV q 12 hours for 6 weeks Methicillin-resistant Staphylococcus aureus (MRSA) has to be treated with an antibiotic that MRSA is sensitive to, such as vancomycin. Vancomycin is a glycopeptide antibiotic typically used to treat gram-positive bacteria. Patients with acute osteomyelitis need at least 4-6 weeks of antibiotic therapy. Studies have also been done on animals that show optimal treatment length to be 4-6 weeks in duration.
A 50-year-old obese woman presents with severe left knee pain. She states the pain began about 8 months ago but has grown significantly worse in the last 3 months. The patient denies any trauma or event that initiated the pain. She notes stiffness in the knee first thing in the morning that lasts around 5-10 minutes. The knee pain is worsened with activity and is relieved with rest. The patient's medication list includes lisinopril 10 mg once daily for high blood pressure. She has a documented medication allergy to acetaminophen; she states this makes her break out in hives. Physical examination findings reveal a Caucasian female with a BMI of 40. There is limited range of motion of the left knee and severe crepitus. Question Highlights Considering the most likely diagnosis for this patient, what lifestyle modification would be most beneficial in relieving her symptoms? 1 Bed rest 2 Blood pressure control 3 High impact activity 4 Screening labs 5 Weight loss
Correct Answer: Weight loss This patient is most likely suffering from degenerative joint disease, also known as osteoarthritis (OA). OA is the most common form of knee arthritis. Common characteristics of OA include onset after 40 (usually in patients over 55) along with obesity; typically, these patients are genetically predisposed. Other components seen in patients with OA initially have an insidious onset of pain that will rapidly progress. Characteristics of this pain will be soon recognized as being exacerbated by activity and relieved at rest. Morning stiffness is common in patients with OA, although it will resolve within a 30 minute time period. Common symptoms patients with OA experience include buckling or giving way of that affected joint due to the bony areas impinging upon each other. Physical examination findings include limited range of motion of that affected joint and crepitus; the patient above has both. Patients may also have tenderness to palpation of the joint, joint effusion, or even palpable osteophytes. Changes that occur in the articular cartilage due to this pathology are irreversible. Conservative treatment is what should be initiated first. This should include activity modification, therapeutic exercises, weight loss, proper footwear, and avoidance of high impact activities. Use of a cane may also be mechanically advantageous for the patient, especially if it is used in the hand opposite the affected side. Knee braces or sleeves may also be utilized and provide minor subjective pain relief. Initial medication intervention of choice should be oral acetaminophen, topical capsaicin, or a traditional NSAID.
A 66-year-old woman presents for a health maintenance visit. She reports no issues and has no history of chronic illness except postmenopausal osteoporosis, with a bone mineral density >3 standard deviations below the mean. She takes supplemental calcium and is being treated with denosumab IM every 6 months. Her only recent fracture was a compression fracture at T-6, diagnosed 5 months ago. Although she had previously enjoyed tennis, dancing, and gardening, she has drastically reduced her activity for fear of suffering further fractures. She asks if she should resume any sort of regular physical activity. Past medical history is otherwise unremarkable except for anemia, which is now resolved. Vital signs are normal, as is the remainder of the examination. Question Highlights What advice would you give? 1 Perform only activities of daily living due to history of fracture. 2 Exercise is contraindicated with osteoporosis. 3 Work toward resuming former exercise program. 4 Begin weight training, but avoid weight-bearing activities. 5 Wait to resume exercise until denosumab is discontinued.
Correct Answer: Work toward resuming former exercise program. Inadequate physical activity contributes to the development of osteoporosis. This patient should work towards resuming her former exercise program. Regular weight-bearing and muscle strengthening exercises are recommended to maintain and improve bone strength; to improve agility, strength, and posture; and to reduce the risk of falls. Such a regimen may also lead to some increase in bone density. Tennis and jogging are weight-bearing activities, to which a muscle strengthening activity such as weight training can be added.
A 36-year-old G1P1001 woman presents with a 4- to 5-week history of pain along her right lateral wrist; the pain worsens when she tries to grasp something. There was no trauma. She gave birth 6 weeks ago and carrying her baby is difficult secondary to pain. She is right-handed and has never had any issues like this before. On exam, there is tenderness and edema over the radial styloid. You then have the patient fully flex her thumb, adduct, and grasp it with that hand. You then place her hand in ulnar deviation, which reproduces the pain described above. Question What is the most likely diagnosis? 1 Rheumatoid arthritis 2 Ganglion cyst 3 Scaphoid fracture 4 de Quervain tenosynovitis 5 Radial nerve entrapment
Correct Answer: de Quervain tenosynovitis de Quervain Tenosynovitis is a tenosynovitis of the abductor pollicis longus and extensor pollicis brevis tendons at the radial styloid process. It is most commonly seen in women 35-55. It can happen when women carry children with their thumb overextended. On exam, there can be tenderness and swelling over the radial styloid. There is also a positive Finkelstein test, which is what is described in the scenario.
A 49-year-old right-hand dominant woman presents with a 2-week history of progressive pain in her right thumb and wrist area. She says that her thumb seems to "stick" in place upon movement. She states that she has never experienced this before. You ask her if she has been using her hands more often than normal, and she tells you she recently began to crochet a sweater for a family member. On physical examination, her pain is markedly exacerbated when she places her thumb into the palm of her hand, and when you passively move her hand in an ulnar direction. Question Highlights What is the most likely diagnosis? 1 Wrist arthritis 2 Scaphoid fracture 3 Dorsal wrist ganglion 4 Thumb arthritis 5 de Quervain's tenosynovitis
Correct Answer: de Quervain's tenosynovitis The clinical picture is suggestive of de Quervain's tenosynovitis. It occurs commonly in middle-aged women, precipitated by repetitive use of the thumb. The inflammation thickens the tendon sheath abductor pollicis longus and extensor brevis on the thumb side of the wrist. This can cause pain, swelling, and may cause the thumb to lock in place upon movement. The Finkelstein test (full flexion of thumb into palm followed by ulnar deviation of the wrist) will produce pain and is diagnostic of de Quervain's tenosynovitis.
A 32-year-old woman with no significant past medical history presents with a 3-month history of right anterior knee pain described as a dull and aching pain that is "right under the kneecap." Provocative activities include bending movements, descending stairs, and performing squatting maneuvers. Pain is relieved during rest. She notes that she loves the outdoors and her hobbies include running and hiking; her symptoms began following a run. She denies a history of falls, prior surgeries or instrumentation, fever, chills, malaise, myalgias, changes in weight, joint swelling, skin changes or rashes, or other joint pains. Her physical exam is normal with the exception of a tender undersurface of the patella, with crepitus upon passive range of motion of the right knee. There is abnormal patellar tracking upon right knee flexion and apprehension of the patient upon passive manipulation of the patella. Additionally, there is a positive patellar grind test. McMurray's, Lachman, the anterior and posterior drawer, Apley's compression and distraction tests, and varus/valgus tests are all negative. There is no joint line tenderness, effusion, or restriction of range of motion of the right knee. What is the most likely diagnosis? 1 Right knee meniscal tear 2 Tear of the medial collateral ligament (MCL) 3 Anterior cruciate ligament (ACL) tear 4 Iliotibial band syndrome 5 Patellofemoral pain syndrome
This patient exhibits signs and symptoms of patellofemoral pain syndrome, also known as chondromalacia patellae, miserable malalignment syndrome, and runner's knee. This syndrome describes any pain involving the patellofemoral joint. The pain affects any or all of the anterior knee structures, including the medial and lateral aspects of the patella, as well as the quadriceps and patellar tendon insertions. The patella engages the femoral trochlear groove with approximately 30° of knee flexion. Forces on the patellofemoral joint increase up to 3 times body weight as the knee flexes to 90° (e.g., climbing stairs), and 5 times body weight when going into full knee flexion (e.g., squatting). Abnormal patellar tracking during flexion can lead to abnormal articular cartilage wear and pain. When the patient has ligamentous hyperlaxity, the patella can sublux out of the groove, usually laterally. Patellofemoral pain is also associated with muscle strength and flexibility imbalances, as well as altered hip and ankle biomechanics.